Você está na página 1de 26

UNION MANUFACTURING CO., INC. VS.

PHILIPPINE GUARANTY
CO., INC.

vs.
THE CENTURY INSURANCE CO., LTD., defendant-appellant.

47 SCRA 271 (G.R. NO. L-27932)

G.R. No. L-22738

December 2, 1924

Facts:
OCTOBER 30, 1972
Petitioner:

Republic Bank

Respondent:

Philippine Guaranty Co.. Inc.

FACTS:
On January 12, 1962, the Union Manufacturing Co., Inc. obtained certain
loans from the Republic Bank in the total sum of 415,000.00. To
secure the payment thereof, UMC executed real and chattel mortgage
on certain properties.
The Republic Bank procured from the defendant Philippine Guaranty
Co., Inc. an insurance coverage on loss against fire for 500,000.00
over the properties of the UMC, as described in defendants cover note
dated September 25, 1962, with the annotation that loss or damage, if
any, under said cover note is payable to Republic Bank as its interest
may appear, subject however to the printed conditions of said
defendants Fire Insurance Policy Form.
On September 6, 1964, a fire occurred in the premises of UMC and on
October 6, 1964, UMC filed its fire claim with the PGC Inc., thru its
adjuster, H.H. Bayne Adjustment Co., which was denied by said
defendant in its letter dated November 26, 1964 on the following ground:
Policy Condition No. 3 and/or the Other Insurance Clause of the policy
was violated because you did not give notice to us of the other insurance
which you had taken from New India for 80,000.00. Sincere Insurance
for 25,000.00 and Manila Insurance for 200,000.00 with the result
that these insurances of which we became aware of only after the fire,
were not endorsed on our policy.
ISSUE:
Whether Republic Bank can recover.
HELD:
Without deciding- whether notice of other insurance upon the same
property must be given in writing, or whether a verbal notice is sufficient
to render an insurance valid which requires such notice, whether oral or
written, we hold that in the absolute absence of such notice when it is
one of the conditions specified in the fire insurance policy, the policy is
null and void. (Santa Ana vs. Commercial Union Ass. Co., 55 Phil. 128).
If the insured has violated or failed to perform the conditions of the
contract, and such a violation or want of performance has not been
waived by the insurer, then the insured cannot recover. Courts are not
permitted to make contracts for the parties. The functions and duty of the
courts consist simply in enforcing and carrying out the contracts actually
made.
While it is true, as a general rule, that contracts of insurance are
construed most favorably to the insured, yet contracts of insurance, like
other contracts, are to be construed according to the sense and meaning
of the terms which the parties themselves have used. If such terms are
clear and unambiguous they must be taken and understood in their plain,
ordinary and popular sense.
The annotation then, must be deemed to be a warranty that the property
was not insured by any other policy. Violation thereof entitles the insurer
to rescind. The materiality of non-disclosure of other insurance policies is
not open to doubt.
The insurance contract may be rather onerous, but that in itself does not
justify the abrogation of its express terms, terms which the insured
accepted or adhered to and which is the law between the contracting
parties.

ONG GUAN CAN and THE BANK OF THE PHILIPPINE ISLANDS,


plaintiffs-appellees,

A building of the plaintiff was insured against fire by the defendant in the
sum of P30,000, as well as the goods and merchandise therein
contained in the sum of P15,000. The house and merchandise insured
were burnt early in the morning of February 28, 1923, while the policies
issued by the defendant in favor of the plaintiff were in force. The
appellant contends that under clause 14 of the conditions of the policies,
it may rebuild the house burnt, and although the house may be smaller,
yet it would be sufficient indemnity to the insured for the actual loss
suffered by him. The clause states that: The Company may at its option
reinstate or replace the property damaged or destroyed, or any part
thereof, instead of paying the amount of the loss of damages, or may join
with any other Company or insurers in so doing, but the Company shall
not be bound to reinstate exactly or completely, but only as
circumstances permit and in reasonable sufficient manner, and in no
case shall the Company be bound to expend more in reinstatement that
it would have cost to reinstate such property as it was at the time of the
occurrence of such loss or damage, nor more than the sum insured by
the Company thereon. If the clause is valid it may either rebuild it or pay
it. It must be taken in consideration that the insurance company must
notify the insured which between the 2: either rebuild it or pay it, will be
fulfilled. In which case, the final notice was not given and the rebuilding
of the property would be the better option.
Issue:
Whether or not the Insurance Company should rebuild the house or not?
Ruling of the Court:
It was held that the trial judge very aptly says in his decision: "It would be
an imposition unequitable, as well as unjust, to compel the plaintiff to
accept the rebuilding of a smaller house than the one burnt, with a lower
kind of materials than those of said house, without offering him an
additional indemnity for the difference in size between the two house,
which circumstances were taken into account when the insurance
applied for by the plaintiff was accepted by the defendant." And we may
add: Without tendering either the insured value of the merchandise
contained in the house destroyed, which amounts to the sum of
P15,000." The election alleged by the appellant to rebuild the house
burnt instead of paying the value of the insurance is improper.
G.R. No. L-26827

June 29, 1984

AGAPITO GUTIERREZ, plaintiff-appellee,


vs.
CAPITAL INSURANCE & SURETY CO., INC., defendant-appellant.
Facts:
Capital Insurance & Surety Co., Inc. insured on December 7, 1961 for
one year the jeepney of Agapito Gutierrez against passenger and thirdparty liability. The passenger liability would not exceed P5,000 for any
one person.The policy provides in item 13 that the authorized driver must
be the holder of a valid and subsisting professional driver's license. "A
driver with an expired Traffic Violation Receipt or expired Temporary
Operator's Permit is not considered an authorized driver." On May 29,
1962, the insured jeepney had an accident at Buendia Avenue, Makati,
Rizal. As a result of said accident, a passenger named Agatonico
Ballega fell off the vehicle and died. Teofilo Ventura, the jeepney driver,
was duly licensed for the years 1962 and 1963. However, at the time of
the accident he did not have the license. Instead, he had a carbon copy
of a traffic violation report (summons) issued by a policeman on February
22, 1962, with the notation that he had committed the violation:
"Inattentive to driving (Inv. in accident) at 9:30 a.m., 2-22-62". The
same traffic violation report, which served as a receipt for his license,
required him to report to Branch 8 of the traffic court at the corner of
Arroceros and Concepcion Streets, Manila at nine o'clock in the morning
of March 2, 1962. The TVR would "serve as a temporary operator's
permit for 15 days from receipt hereof". It is indisputable that at the time
of the accident (May 29, 1962), Ventura was holding an "expired
Temporary Operator's Permit." Capital Insurance refused to make any
reimbursement with regard to Guttierez's payment to the widow, hence
he filed on October 14, 1963 in the city court of Manila an action for
specific performance and damages. Insurance Company contended that
paragraph 13 of the policy, already cited, is decisive and controlling in
this case. It plainly provides, and we repeat, that "a driver with an expired

Traffic Violation Receipt or expired Temporary Operator's permit is not


considered an authorized driver within the meaning" of the policy.
Obviously, Ventura was not an authorized driver. His temporary
operator's permit had expired. The expiration bars recovery under the
policy. In liability insurance, "the parties are bound by the terms of the
policy and the right of insured to recover is governed thereby" (44 C.J.S.
934)
Issue:
Whether an insurance covers a jeepney whose driver's traffic violation
report or temporary operator's permit had already expired?
Rulling of the Court:
It was held that the following ruling has persuasive authority with regards
to Insurance:
Insurance; Automobile; When insurer exempt from liability; Case at bar.
The automobile insurance policy sued upon in the instant case
exempts the insurer company from liability for any accident loss, damage
or liability caused, sustained or incurred while the vehicle is being driven
by any person other than an authorized driver.
The policy defines the term 'authorized driver' to be the insured himself
or any person driving on the insured's order or with his permission
provided he is permitted to drive under the licensing laws.

Insurance, excluding its liability to pay claims under the policy in behalf
of "persons who are under the age of sixteen (16) years of age or over
the age of sixty (60) years ..." It is pointed out that the insured being over
sixty (60) years of age when she applied for the insurance coverage, the
policy was null and void, and no risk on the part of the respondent
insurance corporation had arisen therefrom.

The trial court sustained the contention of the private respondent and
dismissed the complaint. It was reasoned out that a policy of insurance
being a contract of adhesion, it was the duty of the insured to know the
terms of the contract he or she is entering into; the insured in this case,
upon learning from its terms that she could not have been qualified
under the conditions stated in said contract, what she should have done
is simply to ask for a refund of the premium that she paid. It was further
argued by the trial court that the ruling calling for a liberal interpretation
of an insurance contract in favor of the insured and strictly against the
insurer may not be applied in the present case in view of the peculiar
facts and circumstances obtaining therein.

In the given case, plaintiff's brother, who was at the wheel at the time of
the collision, did not have a valid license because the one he had
obtained had already expired and had not been renewed as required by
Section 31 of the Motor Vehicle Law. That since he had renewed his
license one week after the accident, it did not cure the delinquency or
revalidate the license which had already expired (Syllabus, Tanco, Jr. vs.
Phil. Guaranty Co., 122 Phil. 709). Wherefore the case is against
Gutierrez.

Issue:

REGINA L. EDILLON, as assisted by her husband, MARCIAL


EDILLON, petitioners-appellants,

Ruling:

Whether or not the acceptance by the private respondent insurance


corporation of the premium and the issuance of the corresponding
certificate of insurance should be deemed a waiver of the exclusionary
condition of overage stated in the said certificate of insurance.

vs.
MANILA BANKERS LIFE INSURANCE CORPORATION and the
COURT OF FIRST INSTANCE OF RIZAL, BRANCH V, QUEZON CITY,
respondents-appellees.

G.R. No. L-34200 September 30, 1982


Facts:

Sometime in April 1969, Carmen Lapuz applied with respondent


insurance corporation for insurance coverage against accident and
injuries. She filled up the blank application form given to her and filed the
same with the respondent insurance corporation. In the said application
form which was dated April 15, 1969, she gave the date of her birth as
July 11, 1904. On the same date, she paid the sum of P20.00
representing the premium for which she was issued the corresponding
receipt signed by an authorized agent of the respondent insurance
corporation.Upon the filing of said application and the payment of the
premium on the policy applied for, the respondent insurance corporation
issued its Certificate. The policy was to be effective for a period of 90
days. On May 31, 1969 or during the effectivity of Certificate of
Insurance, Carmen O. Lapuz died in a vehicular accident. Petitioner
Regina L. Edillon, a sister of the insured and who was the named
beneficiary in the policy, filed her claim for the proceeds of the insurance,
submitting all the necessary papers and other requisites with the private
respondent. Her claim having been denied, Regina L. Edillon instituted
this action in the Court of First Instance.

In resisting the claim of the petitioner, the respondent insurance


corporation relies on a provision contained in the Certificate of

The Supreme Court reversed trial court's decision. The Court ruled that
the age of the insured Carmen Lapuz was not concealed to the
insurance company. Her application for insurance coverage which was
on a printed form furnished by private respondent and which contained
very few items of information clearly indicated her age of the time of filing
the same to be almost 65 years of age. Despite of, it could hardly be
overlooked in the application form, considering its prominence thereon
and its materiality to the coverage applied for, the respondent insurance
corporation received her payment of premium and issued the
corresponding certificate of insurance without question. The accident
which resulted in the death of the insured, a risk covered by the policy,
occurred on May 31, 1969 or FORTY-FIVE (45) DAYS after the
insurance coverage was applied for. There was sufficient time for the
private respondent to process the application and to notice that the
applicant was over 60 years of age and thereby cancel the policy on that
ground if it was minded to do so. If the private respondent failed to act, it
is either because it was willing to waive such disqualification; or, through
the negligence or incompetence of its employees for which it has only
itself to blame, it simply overlooked such fact. Under the circumstances,
the insurance corporation is already deemed in estoppel. It is usually
held that where the insurer, at the time of the issuance of a policy of
insurance, has knowledge of existing facts which, if insisted on, would
invalidate the contract from its very inception, such knowledge
constitutes a waiver of conditions in the contract inconsistent with the
known facts, and the insurer is stopped thereafter from asserting the
breach of such conditions. The law is charitable enough to assume, in
the absence of any showing to the contrary, that an insurance company
intends to execute a valid contract in return for the premium received;
and when the policy contains a condition which renders it voidable at its
inception, and this result is known to the insurer, it will be presumed to
have intended to waive the conditions and to execute a binding contract,
rather than to have deceived the insured into thinking he is insured when
in fact he is not, and to have taken is money without consideration.
THE CAPITAL INSURANCE & SURETY CO., INC., petitioner,

vs.
PLASTIC ERA CO., INC., AND COURT OF APPEALS, respondents.
G.R. No. L-22375 July 18, 1975

Facts:

On December 17, 1960, petitioner Capital Insurance & Surety Co.,


Inc.delivered to the respondent Plastic Era Manufacturing Co., Inc., its
open Fire Policy wherein the former undertook to insure the latter's
building, equipments, raw materials, products and accessories. The
policy expressly provides that if the property insured would be destroyed
or damaged by fire after the payment of the premiums, at anytime
between the 15th day of December 1960 and one o'clock in the
afternoon of the 15th day of December 1961, the insurance company
shall make good all such loss or damage in an amount not exceeding
P100,000.00. When the policy was delivered, Plastic Era failed to pay
the corresponding insurance premium. However, through its duly
authorized representative, it executed promisory note payable within 30
days which was accepted by the respondent. On January 8, 1961, in
partial payment of the insurance premium, Plastic Era delivered to
Capital Insurance, a check for the amount of P1,000.00 postdated
January 16, 1961 payable to the order of the latter and drawn against the
Bank of America. However, the same was dishonored by the bank for
lack of funds. The records show that as of January 19, 1961 Plastic Era
had a balance of P1,193.41 with the Bank of America. Two days after the
insurance premium became due, at about 4:00 to 5:00 o'clock in the
morning, the property insured by Plastic Era was destroyed by fire. In
due time, the latter notified Capital Insurance of the loss of the insured
property by fire and accordingly filed its claim for indemnity. The loss
and/or damage suffered by Plastic Era was estimated to be P283,875.
However, according to the records the same property has been insured
by Plastic Era with the Philamgen Insurance Company for P200,000.00.
In less than a month Plastic Era demanded from Capital Insurance the
payment of the sum of P100,000.00 as indemnity for the loss of the
insured property under Policy but the latter refused for the reason that,
among others, Plastic Era failed to pay the insurance premium.

On August 25, 1961, Plastic Era filed its complaint against Capital
Insurance for the recovery of the sum of P100,000.00 plus P25,000.00
for attorney's fees and P20,000.00 for additional expenses. Capital
Insurance filed a counterclaim of P25,000.00 as and for attorney's fees.
The trial court rendered judgment in favor of the plaintiff and against the
defendant for the sum of P88,325.63 with interest at the legal rate from
the filing of the complaint and to pay the costs. Court of appeals affirmes
lower court's decision. Thus, Capital Insurance elevated the case in the
Supreme Court.
Issue:
Whether or not a contract of insurance has been duly perfected between
the petitioner, Capital Insurance, and respondent Plastic Era.
Ruling: Yes

In clear and unequivocal terms the insurance policy provides that it is


only upon payment of the premiums by Plastic Era that Capital Insurance
agrees to insure the properties of the former against loss or damage in
an amount not exceeding P100,000.00.

The mere delivery of a bill of exchange in payment of a debt does not


immediately effect payment. It simply suspends the action arising from
the original obligation in satisfaction of which it was delivered, until
payment is accomplished either actually or presumptively. Tender of draft
or check in order to effect payment that would extinguish the debtor's
liability should be actually cashed. If the delivery of the check of Plastic
Era to Capital Insurance were to be viewed in the light of the foregoing,
no payment of the premium had been effected, for it is only when the
check is cashed that it is said to effect payment.
Significantly, in the case at bar, the Capital Insurance accepted the
promise of Plastic Era to pay the insurance premium within thirty (30)
days from the effective date of policy. By so doing, it has implicitly agreed
to modify the tenor of the insurance policy and in effect, waived the
provision therein that it would only pay for the loss or damage in case the
same occurs after the payment of the premium. Considering that the
insurance policy is silent as to the mode of payment, Capital Insurance is
deemed to have accepted the promissory note in payment of the
premium. This rendered the policy immediately operative on the date it
was delivered. The view taken in most cases in the United States:
... is that although one of conditions of an insurance policy is that "it shall
not be valid or binding until the first premium is paid", if it is silent as to
the mode of payment, promissory notes received by the company must
be deemed to have been accepted in payment of the premium. In other
words, a requirement for the payment of the first or initial premium in
advance or actual cash may be waived by acceptance of a promissory
note ...
Precisely, this was what actually happened when the Capital Insurance
accepted the acknowledgment receipt of the Plastic Era promising to pay
the insurance premium within thirty (30) days from December 17, 1960.
Hence, when the damage or loss of the insured property occurred, the
insurance policy was in full force and effect. The fact that the check
issued by Plastic Era in partial payment of the promissory note was later
on dishonored did not in any way operate as a forfeiture of its rights
under the policy, there being no express stipulation therein to that effect.
In the absence of express agreement or stipulation to that effect in the
policy, the non-payment at maturity of a note given for and accepted as
premium on a policy does not operate to forfeit the rights of the insured
even though the note is given for an initial premium, nor does the fact
that the collection of the note had been enjoined by the insured in any
way affect the policy.

... If the check is accepted as payment of the premium even though it


turns out to be worthless, there is payment which will prevent forfeiture.

By accepting its promise to pay the insurance premium within thirty (30)
days from the effectivity date of the policy December 17, 1960 Capital
Insurance had in effect extended credit to Plastic Era. The payment of
the premium on the insurance policy therefore became an independent
obligation the non-fulfillment of which would entitle Capital Insurance to
recover. It could just deduct the premium due and unpaid upon the
satisfaction of the loss under the policy. It did not have the right to cancel
the policy for nonpayment of the premium except by putting Plastic Era
in default and giving it personal notice to that effect. This Capital
Insurance failed to do.

... Where credit is given by an insurance company for the payment of the
premium it has no right to cancel the policy for nonpayment except by
putting the insured in default and giving him personal notice....

On the contrary Capital Insurance had accepted a check for P1,000.00


from Plastic Era in partial payment of the premium on the insurance
policy. Although the check was due for payment on January 16, 1961
and Plastic Era had sufficient funds to cover it as of January 19, 1961,
Capital Insurance decided to hold the same for thirty-five (35) days
before presenting it for payment. Having held the check for such an
unreasonable period of time, Capital Insurance was estopped from
claiming a forfeiture of its policy for non-payment even if the check had
been dishonored late.

IGNACIO SATURNINO, in his own behalf and as the JUDICIAL


GUARDIAN OF CARLOS SATURNINO
vs.
THE PHILIPPINE AMERICAN LIFE INSURANCE COMPANY

G.R. No. L-16163

February 28, 1963

Facts:

The policy sued upon is one for 20-year endowment non-medical


insurance. This kind of policy dispenses with the medical examination of
the applicant usually required in ordinary life policies. However, detailed
information is called for in the application concerning the applicant's
health and medical history. The written application in this case was
submitted by Saturnino to appellee on November 16, 1957, witnessed by
appellee's agent Edward A. Santos. The policy was issued on the same
day, upon payment of the first year's premium of P339.25. On
September 19, 1958 Saturnino died of pneumonia, secondary to
influenza. Appellants here, who are her surviving husband and minor
child, respectively, demanded payment of the face value of the policy.
The claim was rejected and this suit was subsequently instituted. Two
months prior to the issuance of the policy or on September 9, 1957,
Saturnino was operated on for cancer, involving complete removal of the
right breast, including the pectoral muscles and the glands found in the
right armpit. She stayed in the hospital for a period of eight days, after
which she was discharged, although according to the surgeon who
operated on her she could not be considered definitely cured, her
ailment being of the malignant type. Notwithstanding the fact of her
operation Estefania A. Saturnino did not make a disclosure thereof in her
application for insurance. On the contrary, she stated therein that she did
not have, nor had she ever had, among other ailments listed in the
application, cancer or other tumors; that she had not consulted any
physician, undergone any operation or suffered any injury within the
preceding five years; and that she had never been treated for nor did she
ever have any illness or disease peculiar to her sex, particularly of the
breast, ovaries, uterus, and menstrual disorders. The application also
recites that the foregoing declarations constituted "a further basis for the
issuance of the policy."

Plaintiffs, now appellants, filed this action in the Court of to recover the
sum of P5,000.00, corresponding to the face value of an insurance policy
issued by defendant on the life of Estefania A. Saturnino, and the sum of
P1,500.00 as attorney's fees.
Both the complaint and the counterclaim were dismissed by the trial
court; but appellants were declared entitled to the return of the premium
already paid; plus interest at 6% up to January 8, 1959, when a check for
the corresponding amount - P359.65 - was sent to them by appellee.

ISSUE:
Whether or not the insured made such false representations of material
facts as to avoid the policy.

RULING:

The Insurance Law (Section 30) provides that "materiality is to be


determined not by the event, but solely by the probable and reasonable
influence of the facts upon the party to whom the communication is due,
in forming his estimate of the proposed contract, or in making his
inquiries." It seems to be the contention of appellants that the facts
subject of the representation were not material in view of the "nonmedical" nature of the insurance applied for, which does away with the
usual requirement of medical examination before the policy is issued.
The contention is without merit. If anything, the waiver of medical
examination renders even more material the information required of the
applicant concerning previous condition of health and diseases suffered,
for such information necessarily constitutes an important factor which the
insurer takes into consideration in deciding whether to issue the policy or
not. It is logical to assume that if appellee had been properly apprised of
the insured's medical history she would at least have been made to
undergo medical examination in order to determine her insurability.

In the case of Kasprzyk v. Metropolitan Insurance Co., 140 N.Y.S. 211,


214, it was held:
Moreover, if it were the law that an insurance company could not depend
a policy on the ground of misrepresentation, unless it could show actual
knowledge on the part of the applicant that the statements were false,
then it is plain that it would be impossible for it to protect itself and its
honest policyholders against fraudulent and improper claims. It would be
wholly at the mercy of any one who wished to apply for insurance, as it
would be impossible to show actual fraud except in the extremest cases.
It could not rely on an application as containing information on which it
could act. There would be no incentive to an applicant to tell the truth.
In this jurisdiction a concealment, whether intentional or unintentional,
entitles the insurer to rescind the contract of insurance, concealment
being defined as "negligence to communicate that which a party knows
and ought to communicate" (Sections 24 & 26, Act No. 2427). In the
case of Argente v. West Coast Life Insurance Co., 51 Phil. 725, 732, this
Court said, quoting from Joyce, The Law of Insurance, 2nd ed., Vol. 3:

"The basis of the rule vitiating the contract in cases of concealment is


that it misleads or deceives the insurer into accepting the risk, or
accepting it at the rate of premium agreed upon. The insurer, relying
upon the belief that the assured will disclose every material fact within
his actual or presumed knowledge, is misled into a belief that the
circumstance withheld does not exist, and he is thereby induced to
estimate the risk upon a false basis that it does not exist."

Supreme Court affirmed the decision of the lower court.

SUN INSURANCE OFFICE, LTD


vs.
COURT OF APPEALS and EMILIO TAN
G.R. No. 89741 March 13, 1991
Facts:

On August 15, 1983, herein private respondent Emilio Tan took from
herein petitioner a P300,000.00 property insurance policy to cover his
interest in the electrical supply store of his brother housed in a building in
Iloilo City. Four (4) days after the issuance of the policy, the building was
burned including the insured store. On August 20, 1983, Tan filed his
claim for fire loss with petitioner, but on February 29, 1984, petitioner
wrote Tan denying the latter's claim. On April 3, 1984, Tan wrote
petitioner, seeking reconsideration of the denial of his claim. On
September 3, 1985, Tan's counsel wrote the Insurer inquiring about the
status of his April 3, 1984 request for reconsideration. Petitioner
answered the letter on October 11, 1985, advising Tan's counsel that the
Insurer's denial of Tan's claim remained unchanged, enclosing copies of
petitioners' letters of February 29, 1984 and May 17, 1985 (response to
petition for reconsideration). On November 20, 1985, Tan filed Civil Case
with the Regional Trial Court but petitioner filed a motion to dismiss on
the alleged ground that the action had already prescribed. Said motion
was denied in an order dated November 3, 1987; and petitioner's motion
for reconsideration was also denied in an order dated January 14, 1988.

be brought by the insured while the evidence as to the origin and cause
of destruction have not yet disappeared.
In enunciating the above-cited principle, this Court had definitely settled
the rationale for the necessity of bringing suits against the Insurer within
one year from the rejection of the claim. The contention of the
respondents that the one-year prescriptive period does not start to run
until the petition for reconsideration had been resolved by the insurer,
runs counter to the declared purpose for requiting that an action or suit
be filed in the Insurance Commission or in a court of competent
jurisdiction from the denial of the claim. To uphold respondents'
contention would contradict and defeat the very principle which this
Court had laid down. Moreover, it can easily be used by insured persons
as a scheme or device to waste time until any evidence which may be
considered against them is destroyed.
It is apparent that Section 27 of the insurance policy was stipulated
pursuant to Section 63 of the Insurance Code, which states that:

Petitioner went to the Court of Appeals and sought the nullification of the
said Nov. 3, 1987 and January 14, 1988 orders, but the Court of Appeals,
in its June 20, 1989 decision denied the petition and held that the court a
quomay continue until its final termination. A motion for reconsideration
was filed, but the same was denied by the Court of Appeals in its
resolution of August 22, 1989.

Sec. 63. A condition, stipulation or agreement in any


policy of insurance, limiting the time for commencing
an action thereunder to a period of less than one
year from the time when the cause of action
accrues, is void.

Hence, petitioners elevated the case to the Supreme Court.


ISSUE:
Whether or not the filing of a motion for reconsideration interrupts the 12
months prescriptive period to contest the denial of the insurance claim.
RULING: NO
While it is a cardinal principle of insurance law that a policy or contract of
insurance is to be construed liberally in favor of the insured and strictly
against the insurer company, yet, contracts of insurance, like other
contracts, are to be construed according to the sense and meaning of
the terms which the parties themselves have used. If such terms are
clear and unambiguous, they must be taken and understood in their
plain, ordinary and popular sense.
Condition 27 of the Insurance Policy, which is the subject of the
conflicting contentions of the parties, reads:
27. Action or suit clause If a claim be made and rejected and an
action or suit be not commenced either in the Insurance Commission or
in any court of competent jurisdiction within twelve (12) months from
receipt of notice of such rejection, or in case of arbitration taking place as
provided herein, within twelve (12) months after due notice of the award
made by the arbitrator or arbitrators or umpire, then the claim shall for all
purposes be deemed to have been abandoned and shall not thereafter
be recoverable hereunder.
As the terms are very clear and free from any doubt or ambiguity
whatsoever, it must be taken and understood in its plain, ordinary and
popular sense pursuant to the above-cited principle laid down by this
Court.

DIONISIA, EULOGIO, MARINA, GUILLERMO and NORBERTO all


surnamed
GUINGON, plaintiffs-appellees,
vs.
ILUMINADO DEL MONTE, JULIO AGUILAR and CAPITAL
INSURANCE
and
SURETY
CO.,
INC., defendants.
CAPITAL INSURANCE and SURETY CO., INC., defendant-appellant.
G.R. No. L-22042

August 17, 1967

FACTS:
Julio Aguilar owned and operated several jeepneys in the City of
Manila. . He entered into a contract with the Capital Insurance & Surety
Co., Inc. insuring the operation of his jeepneys against accidents with
third-party liability. Insurance policy was executed by the Capital
Insurance & Surety Co., Inc., it contains the following provision:
Section II LIABILITY TO THE PUBLIC
1. The Company, will, subject to the limits of liability, indemnify
the Insured in the event of accident caused by or arising out of
the use of the Motor Vehicle/s or in connection with the loading
or unloading of the Motor Vehicle/s, against all sums including
claimant's costs and expenses which the Insured shall become
legally liable to pay in respect of:
a. death of or bodily injury to any person

Respondent Tan, in his letter addressed to the petitioner insurance


company dated April 3, 1984, admitted that he received a copy of the
letter of rejection on April 2, 1984. Thus, the 12-month prescriptive period
started to run from the said date of April 2, 1984, for such is the plain
meaning and intention of Section 27 of the insurance policy.
It is also important to note the principle laid down by this Court in the
case of Ang v. Fulton Fire Insurance Co., (2 SCRA 945 [1961]), to wit:
The condition contained in an insurance policy that claims must be
presented within one year after rejection is not merely a procedural
requirement but an important matter essential to a prompt settlement of
claims against insurance companies as it demands that insurance suits

b. damage to property
During the effectivity of such insurance policy, Iluminado del Monte, one
of the drivers of the jeepneys operated by Aguilar, bumped with the
jeepney abovementioned one Gervacio Guingon who had just alighted
from another jeepney and as a consequence the latter died some days
thereafter. An information for homicide thru reckless imprudence was
filed against Iluminado del Monte, who pleaded guilty. A penalty of four
months imprisonment was imposed on him.
The heirs of Gervacio Guingon filed an action for damages to be paid to
them jointly and severally by the defendants, driver Iluminado del Monte,

owner and operator Julio Aguilar, and the Capital Insurance & Surety
Co., Inc.
The Court of First Instance of Manila rendered its judgment in favor of
the heirs of the deceased holding the defendants jointly liable for
damages.
ARGUMENT:
The case was appealed to the Court of Appeals contending that "no
action" clause in the policy closes the avenue to any third party which
may be injured in an accident wherein the jeepney of the insured might
have been the cause of the injury of third persons, alleging the freedom
of contracts. Will the mere fact that such clause was agreed upon by the
parties in an insurance policy prevail over the Rules of Court which
authorizes the joining of parties plaintiffs or defendants.
ISSUE:
Whether or not the plaintiffs being a third party can jointly sue the insurer
and the insured.
RULING:
Yes. The policy in the present case is one for indemnity against liability;
from the fact then that the insured is liable to the third person, such third
person is entitled to sue the insurer. And the "no action" clause in the
policy of insurance cannot prevail over the Rules of Court provision
aimed at avoiding multiplicity of suits.
The Rules of Court provides that; two causes of action are connected
with each other, or grow out of the same transaction, they may be
properly joined, and in such suit all parties against whom the plaintiff
asserts a common or an alternative liability may be joined as defendants.
. . . Even if appellants had presented any plea in abatement as to joinder
of damages arising from a tort with those arising from a contract, it could
not, under the facts of this case, be sustained, for the rule is that a suit
may include an action for breach of contract and one for tort, provided
they are connected with each other or grew out of the same transaction.
In the case at bar, Sec. 5 of Rule 2 on "Joinder of causes of action" and
Sec. 6 of Rule 3 on "Permissive joinder of parties" cannot be
superseded, at least with respect to third persons not a party to the
contract, as herein, by a "no action" clause in the contract of insurance.

On the basis of the evidence presented by the Advincula spouses,


judgment was rendered by the trial court. First Insurance received a copy
of the decision and upon motion of the Advincula spouses, the decision
was amended, which, in addition to the damages granted in the original
decision, awarded damages to Silverio Blanco.
First Insurance filed a petition for relief from judgment in the same case.
But the Court denied the said petition for relief from judgment. The
Advincula spouses moved for the execution of judgment but the First
Insurance filed a petition for relief from the order of execution and
judgment with preliminary injunction but was denied by the court. The
motion for reconsideration was likewise denied.
ARGUMENT:
It is the contention of the petitioner that the Advincula spouses have no
cause of action against it. As parents of the victim, they may proceed
against the driver, Silverio Blanco on the basis of the provisions of the
New Civil Code. However, they have no cause of action against First
Insurance, because they are not parties to the insurance contract.
It is settled that where the insurance contract provides for indemnity
against liability to a third party, such third party can directly sue the
insurer. The liability of the insurer to such third person is based on
contract while the liability of the insured to the third party is based on tort.
ISSUE:
Whether or not Advincula spouses can directly sue the insurer (First
Insurance).
RULING:
Yes. The injured for whom the contract of insurance is intended can sue
directly the insurer. The general purpose of statutes enabling an injured
person to proceed directly against the insurer is to protect injured
persons against the insolvency of the insured who causes such injury,
and to give such injured person a certain beneficial interest in the
proceeds of the policy, and statutes are to be liberally construed so that
their intended purpose may be accomplished. It has even been held that
such a provision creates a contractual relation which inures to the benefit
of any and every person who may be negligently injured by the named
insured as if such injured person were specifically named in the policy.

Wherefore, the judgment appealed from is affirmed.

First Insurance cannot evade its liability as insurer by hiding under the
cloak of the insured. Its liability is primary and not dependent on the
recovery of judgment from the insured.

FIRST INTEGRATED BONDING & INSURANCE COMPANY,


INC., petitioner,
vs.
HON. HAROLD M. HERNANDO, VICTORINO ADVINCULA, ROMANA
ADVINCULA, SILVERIO BLANCO & THE SHERIFF OF MANILA and
his DEPUTY SHERIFFS, respondents.

Compulsory Motor Vehicle Liability Insurance (third party liability or TPL)


is primarily intended to provide compensation for the death or bodily
injuries suffered by innocent third parties or passengers as a result of a
negligent operation and use of motor vehicles. The victims and/or their
dependents are assured of immediate financial assistance, regardless of
the financial capacity of the motor vehicle owners. The insurer's liability
accrues immediately upon the occurrence of the injury or event upon
which the liability depends, and does not depend on the recovery of
judgment by the injured party against the insured.

G.R. No. L-51221 July 31, 1991

Wherefore, the petition is dismissed.


FACTS:

VIRGINIA A. PEREZ, petitioner, vs. COURT OF APPEALS and BF


LIFEMAN INSURANCE CORPORATION, respondents.

Silverio Blanco was the owner of a passenger jeepney which he insured


against liabilities for death and injuries to third persons with First
Integrated Bonding and Insurance Company, Inc. (First Insurance).

G.R. No. 112329. January 28, 2000

The said jeepney driven by Blanco himself bumped a five-year old child,
Deogracias Advincula, causing the latter's death.Thereafter, a complaint
for damages was brought by the child's parents, the Advincula spouses,
against Silverio Blanco. First Insurance was also impleaded in the
complaint as the insurer.

FACTS:
Primitivo Perez had been insured with BF Lifeman Insurance
Corporation since 1980 for P20,000. In October 1987 , an agent of
Lifeman Insurance, Rodolfo Lalog, visited Perez in Quezon and

convinced him to apply for additional insurance coverage of P50,000, to


avail of the ongoing promotional discount of P400 if the premium were
paid annually.
Primitivo accomplished an application form for the additional insurance
coverage. Virginia Perez, his wife, paid P2,075 to Lalog. The receipt
issued by Lalog indicated the amount receives as deposit.
Unfortunately, Lalog lost the application form accomplished by Perez and
so on, he asked the latter to fill up another application form. Sometime in
1987, Perez was made to undergo the required medical examination,
which he passed.
Lalog forwarded the application for additional insurance of Perez,
together with all its supporting papers, to the office of BF Lifeman
Insurance in Quezon which was supposed to forward the papers to the
Manila Office.
On November 25, 1987, Perez died while he was riding a banca which
capsized during a storm.
At the time of his death, his application papers for the additional
insurance were still in the Quezon Office. Lalog testified that when he
went to follow up the papers, he found them still in the Quezon office and
so he personally brought the papers to the Manila office. It was only on
November 27, 1987 that the said papers were received in Manila.
Without knowing that Perez died on November 25, 1987, BF Lifeman
Insurance Corporation approved the application and issued the
corresponding policy for P50,000 on December 2, 1987.
Virginia went to Manila to claim the benefits under the insurance policies
of the deceased. She was paid P40,000 under the first insurance policy
for P20,000 (double indemnity in case of accident) but the insurance
company refused to pay the claim under the additional policy coverage
of P50,000, the proceeds of which amount to P150,000 in view of a triple
indemnity rider on the insurance policy.

Insurance is a contract whereby, for a stipulated consideration, one party


undertakes to compensate the other for loss on specific subject by
specific perils. A contract, on the other hand, is a meeting of the minds
between two persons whereby one binds himself, with respect to the
other to give something or to render some service.
Consent must be manifested by the meeting of the offer and the
acceptance upon the thing and the cause which are to constitute the
contract. The offer must be certain and the acceptance absolute. When
Primitivo filed an application for insurance, paid the P2,075 and
submitted the results of his medical examination, his application was
subject to the acceptance of the insurance company.
The perfection of the contract of insurance between the deceased and
the respondent insurance company was further conditioned upon
compliance with the following requisites stated in the application form:
there shall be no contract of insurance unless and until a policy
issued on this application and that the said policy shall not take
effect until the premium has been paid and the policy delivered to
and accepted by me/us in person while I/we,am/are in good health.
The assent of respondent insurance company therefore was not given
when it merely received the application form and all the requisite
supporting papers of the applicant. Its assent was given when it issues a
corresponding policy to the applicant. Under the abovementioned
provision, it is only when the applicant pays the premium and receives
and accepts the policy while he is in good health that the contract of
insurance is deemed to have been perfected.
It is not disputed, however, that Primitivo died on November 25, 1987,
his application papers for additional insurance coverage were still with
the branch office in Quezon and it was only two days later, when Lalog
personally delivered the application papers to the head office in Manila.
Consequently, there was absolutely no way the acceptance of the
application could have been communicated to the applicant for the latter
to accept inasmuch as the applicant at the time was already dead.
Wherefore, the decision of the Court of Appeals is hereby affirmed.

In its letter to Virginia , the insurance company maintained that the


insurance for P50,000 had not been perfected at the time of the death of
Primitivo Perez. Consequently, the insurance company refunded the
amount of P2,075 which Virginia had paid.
ARGUMENT:
Lifeman Insurance filed for the rescission and declaration of nullity
contending that the insurance policy in question was not yet perfected,
hence, the wife cannot claim under said unperfected contract of
insurance. Perez, on the other hand, averred that the deceased had
fulfilled all his prestations under the Contract and all the elements of a
valid contract are present.
Trial Court ruled in favor of Virginia Perez, but on appeal the decision
was reversed by the Court of Appeals, hence this petition.

ISSUE:
Whether or not there was a perfected additional insurance contract.

RULING:
No. the contract was not perfected.
A contract of insurance, like all other contracts, must be assented to by
both parties, either in person or through their agents and so long as an
application for insurance has not been either accepted or rejected, it is
merely a proposal or an offer to make a contract.

LUZ PINEDA, MARILOU MONTENEGRO, VIRGINIA ALARCON, DINA


LORENA AYO, CELIA CALUMBAG and LUCIA LONTOK, petitioners,
vs.
HON. COURT OF APPEALS and THE INSULAR LIFE ASSURANCE
COMPANY, LIMITED, respondents.
G.R. No. 105562 September 27, 1993

FACTS:
The petitioners were the complainants in an administrative complaint
against private respondent Insular Life Assurance Company, Ltd. (Insular
Life), which was filed with the Insurance Commission. They prayed that
Insular Life "be ordered to pay the claimants their insurance claims" and
that "proper sanctions/penalties be imposed on" it "for its deliberate,
feckless violation of its contractual obligations to the complainants, and
of the Insurance Code.
Insular Life's motion to dismiss the complaint on the ground that "the
claims of complainants are all respectively beyond the jurisdiction of the
Insurance Commission as provided in Section 416 of the Insurance
Code." Having been denied it filed its answer, thereafter, hearings were
conducted.The Commission rendered its decision in favor of the
complainants.
Insular Life appealed the decision seeking that the appellate court to
reverse the decision because the Insurance Commission:

(a) had no jurisdiction over the case considering that the claims
exceeded
P100,000.00,
(b) erred in holding that the powers of attorney relied upon by Insular Life
were insufficient to convey absolute authority to Capt. Nuval to demand,
receive and take delivery of the insurance proceeds pertaining to the
petitioners, (c) erred in not giving credit to the version of Insular Life that
the power of attorney supposed to have been executed in favor of the
Alarcons
was
missing,
and
(d) erred in holding that Insular Life was liable for violating Section 180 of
the Insurance Code for having released to the surviving mothers the
insurance proceeds pertaining to the beneficiaries who were still minors
despite the failure of the former to obtain a court authorization or to post
a bond.
The Court of Appeals rendered its decision with modification. Hence this
petition.

DEVELOPMENT INSURANCE CORPORATION vs IAC


G.R. No. 71360 July 16, 1986
(case # 37)
FACTS

a fire occurered in the building of the private respondent


Philippine Union Realty Development Corporation and it sued for
recovery of damages from petitioner on the basis of an insurance
contract between them. Initially, petitioner failed to answer on the said
claims and was declared in default prompting the court to decide in favor
of private respondents

ARGUMENT:
The Insurance Commission had jurisdiction over the case on the ground
that although some of the claims exceed P100,000.00, the petitioners
had asked for administrative sanctions against Insular Life which are
within the Commission's jurisdiction to grant; hence, "there was merely a
misjoinder of causes of action . . . and, like misjoinder of parties, it is not
a ground for the dismissal of the action as it does not affect the other
reliefs prayed for."
When the officers of respondent-appellant read these written powers,
they must have assumed Capt. Nuval indeed had authority to collect the
insurance proceeds in behalf of the beneficiaries who duly affixed their
signatures therein. The written power is specific enough to define the
authority of the agent to collect any sum of money pertaining to the
sinking of the fatal vessel. Respondent-appellant interpreted this power
to include the collection of insurance proceeds in behalf of the
beneficiaries concerned. We believe this is a reasonable interpretation
even by an officer of respondent-appellant unschooled in the law. Had
respondent appellant, consulted its legal department it would not have
received a contrary view. There is nothing in the law which mandates a
specific or special power of attorney to be executed to collect insurance
proceeds.
ISSUE:

ARGUMENTS FOR THE PETITIONER

Petitioner refuses to pay the whole claim of the insured


becuase at the time the fire occured the building was worth Php
5,800,000 the private respondent should be considered its own insurer
for the difference between that amount and the face value of the policy
and should share pro rata the loss sustained. accordingly, the private
respondent is entitled to an indemnity of only Php 67,629, the rest of the
loss to be shouldered by it alone. In support of this contention, petitioner
cites condition 17 of the policy which provides

"If the property hereby insured shall, at the breaking out of any
fire, be collectively of greater value than the sum insured thereon then
the insured shall be considered as being his own insurer for the
difference, and shall bear a ratable proportion of the loss accordingly.
Every item, if more than one, of the policy shall be separately subject to
this condition"

Whether or not the parents of the minors have a right in the policy in
behalf of their minor children.
ARGUMENTS FOR THE RESPONDENT
RULING:
Yes. The Insurance Code expressly provides that; In the absence of a
judicial guardian, the father, or in the latter's absence or incapacity, the
mother, of any minor, who is an insured or a beneficiary under a contract
of life, health or accident insurance, may exercise, in behalf of said
minor, any right under the policy, without necessity of court authority or
the giving of a bond, where the interest of the minor in the particular act
involved does not exceed twenty thousand pesos. Such a right, may
include, but shall not be limited to, obtaining a policy loan, surrendering
the policy, receiving the proceeds of the policy, and giving the minor's
consent to any transaction on the policy.
As amended by the Family Code which grants the father and mother
joint legal guardianship over the property of their unemancipated
common child without the necessity of a court appointment; however,
when the market value of the property or the annual income of the child
exceeds P50,000.00, the parent concerned shall be required to put up a
bond in such amount as the court may determine.
It is clear from the said Article that regardless of the value of the
unemancipated common child's property, the father and mother ipso
jure become the legal guardian of the child's property. However, if the
market value of the property or the annual income of the child exceeds
P50,000.00, a bond has to be posted by the parents concerned to
guarantee the performance of the obligations of a general guardian.
Wherefore, the instant petition is GRANTED. The Decision of the
Insurance Commission is reinstated.

Philippine Union Realty Development contends that it


should be paid for the value of the building at the time when the fire
occured and should not be limited from the value of the policy because
the insurance contract entered is an open policy.

ISSUE

Whether the amount of indemnity due to private respondent is based on


the value of the property at the time of loss.

RULING OF THE COURT

Yes. As defined in the aforestated provision, which is now


Section 60 of the Insurance Code, "an open policy is one in which the
value of the thing insured is not agreed upon but is left to be ascertained
in case of loss. " This means that the actual loss, as determined, will

represent the total indemnity due the insured from the insurer except
only that the total indemnity shall not exceed the face value of the policy.
ISSUE

DISCUSSION
Whether the commisioner has jurisdiction over the subject matter of the
complaint filed by the private respondent
The court in ruling that private respondent is entitled to an
indemnity if Php 508,867 instead of Php 67,629 as assessed by
petitioner noted that the building at the time of loss is still under
construction and not completed, furthermore it observed that the
insurance contract (policy RY/F-082) entered was an open policy based
from the contract itself

"This is an open policy as defined in Section 57 of the


Insurance Act. In the event of loss, whether total or partial, it is
understood that the amount of the loss shall be subject to appraisal and
the liability of the company, if established, shall be limited to the actual
loss, subject to the applicable terms, conditions, warranties and clauses
of this Policy, and in no case shall exceed the amount of the policy"
PHILIPPINE AMERICAN LIFE INSURANCE COMPANY VS. ANSALDO
G.R. NO. 76452 JULY 26, 1994
(Case # 38)

RULING OF THE COURT

No. the quasi-judicial power of the insurance commissioner


under section 416 of the insurance code which states that "The
Commissioner shall have the power to adjudicate claims and complaints
involving any loss, damage or liability for which an insurer may be
answerable under any kind of policy or contract of insurance, or for
which such insurer may be liable under a contract of suretyship, or for
which a reinsurer may be used under any contract or reinsurance it may
have entered into, or for which a mutual benefit association may be held
liable under the membership certificates it has issued to its members,
where the amount of any such loss, damage or liability, excluding
interest, costs and attorney's fees, being claimed or sued upon any kind
of insurance, bond, reinsurance contract, or membership certificate does
not exceed in any single claim one hundred thousand pesos" is limited
by law "to claims and complaints involving any loss, damage or liability
for which an insurer may be answerable under any kind of policy or
contract of insurance, . . ." Hence, this power does not cover the
relationship affecting the insurance company and its agents but is limited
to adjudicating claims and complaints filed by the insured against the
insurance company.

FACTS
DISCUSSION
Private respondent Ramon Paterno filed a letter of complaint
before the respondent commissioner alleging certain problems
encountered by agents, supervisors, managers and public consumers of
the Philippine American Life Insurance Company (Philamlife) as a result
of certain practices by said company. Petitioner asked for clarifications
on what specific acts are complained (Bill of Particulars) of by Paterno.
the commissioner took cognizance of the case and set the hearing.

ARGUMENTS FOR THE PETITIONER

Petitioner contends that the commissioner has no jurisdiction


to entertain such complaint filed by paterno alleging anomalies in the
agency contract between PHILAM and its agents, being cognizable by
the regular courts

the court explained in arriving at this ruling that The Insurance


Code does not have provisions governing the relations between
insurance companies and their agents. It follows that the Insurance
Commissioner cannot, in the exercise of its quasi-judicial powers,
assume jurisdiction over controversies between the insurance
companies and their agents.
in the cases of Great Pacific Life Assurance Corporation v.
Judico, 180 SCRA 445 (1989), and Investment Planning Corporation of
the Philippines v. Social Security Commission, 21 SCRA 904 (1962), an
insurance company may have two classes of agents who sell its
insurance policies: (1) salaried employees who keep definite hours and
work under the control and supervision of the company; and (2)
registered representatives, who work on commission basis. Under the
first category, the relationship between the insurance company and its
agents is governed by the Contract of Employment and the provisions of
the Labor Code, while under the second category, the same is governed
by the Contract of Agency and the provisions of the Civil Code on the
Agency. Disputes involving the latter are cognizable by the regular
courts.

ARGUMENTS FOR THE RESPONDENT


PAN MALAYAN
APPEALS
Private respondent contends that the Insurance Commissioner
has jurisdiction to take cognizance of the complaint in the exercise of its
quasi-judicial powers. The Solicitor General, upholding the jurisdiction of
the Insurance Commissioner, claims that under Sections 414 and 415 of
the Insurance Code, the Commissioner has authority to nullify the
alleged illegal provisions of the Contract of Agency.

INSURANCE

G.R. NO. 81026 APRIL 03, 1990


(CASE # 39)

FACTS

CORPORATION

VS

COUR

OF

On December 10, 1985, PANMALAY filed a complaint for


damages with the RTC of Makati against private respondents Erlinda
Fabie and her driver. PANMALAY averred the following: that it insured a
Mitsubishi Colt Lancer car with plate No. DDZ-431 and registered in the
name of Canlubang Automotive Resources Corporation [CANLUBANG];
that on May 26, 1985, due to the "carelessness, recklessness, and
imprudence" of the unknown driver of a pick-up with plate no. PCR-220,
the insured car was hit and suffered damages in the amount of
P42,052.00; that PANMALAY defrayed the cost of repair of the insured
car and, therefore, was subrogated to the rights of CANLUBANG against
the driver of the pick-up and his employer, Erlinda Fabie; and that,
despite repeated demands, defendants, failed and refused to pay the
claim of PANMALAY.

ARGUMENTS FOR THE PETITIONER

Petitioner contends that pursuant to a motor vehicle insurance policy, it


had indemnified CANLUBANG for the damage to the insured car
resulting from a traffic accident allegedly caused by the negligence of the
driver of private respondent, Erlinda Fabie. PANMALAY contended,
therefore, that its cause of action against private respondents was
anchored upon Article 2207 of the Civil Code, which reads:
If the plaintiffs property has been insured, and he has received indemnity
from the insurance company for the injury or loss arising out of the wrong
or breach of contract complained of, the insurance company shall be
subrogated to the rights of the insured against the wrongdoer or the
person who has violated the contract. . . .

G.R. No. L-18965, October 30, 1964, 12 SCRA 213; Fireman's Fund
Insurance Company v. Jamilla & Company, Inc., G.R. No. L-27427, April
7, 1976, 70 SCRA 323].

DISCUSSION
The court explained that although there are a few recognized exceptions
to this rule. For instance, if the assured by his own act releases the
wrongdoer or third party liable for the loss or damage, from liability, the
insurer's right of subrogation is defeated [Phoenix Ins. Co. of Brooklyn v.
Erie & Western Transport, Co., 117 US 312, 29 L. Ed. 873 (1886);
Insurance Company of North America v. Elgin, Joliet & Eastern Railway
Co., 229 F 2d 705 (1956)]. Similarly, where the insurer pays the assured
the value of the lost goods without notifying the carrier who has in good
faith settled the assured's claim for loss, the settlement is binding on
both the assured and the insurer, and the latter cannot bring an action
against the carrier on his right of subrogation [McCarthy v. Barber
Steamship Lines, Inc., 45 Phil. 488 (1923)]. And where the insurer pays
the assured for a loss which is not a risk covered by the policy, thereby
effecting "voluntary payment", the former has no right of subrogation
against the third party liable for the loss [Sveriges Angfartygs Assurans
Forening v. Qua Chee Gan, G. R. No. L-22146, September 5, 1967, 21
SCRA 12].None of the exceptions are availing in the present case.

ANDRES VS. THE CROWN LIFE INSURANCE COMPANY


G.R. NO. L 10874 JANUARY 28 1958
(CASE # 40)

FACTS

ARGUMENTS FOR THE RESPONDENT

private respondents contends that PANMALAY had no cause


of action against them. They argued that payment under the "own
damage" clause of the insurance policy precluded subrogation under
Article 2207 of the Civil Code, since indemnification thereunder was
made on the assumption that there was no wrongdoer or no third party at
fault.

On April 20, 1952, Rufino D. Andres filed a complaint in the Court of First
Instance of Ilocos Norte against the Crown Life Insurance Company for
the recovery of the amount of P5,000, as the face value of a joint 20-year
endowment insurance policy issued in favor of the plaintiff Rufino D.
Andres and his wife Severa G. Andres on the 13th of February, 1950, by
said insurance company. On Jun 7, 1951, Rufino Andres presented his
death claim as survivor-beneficiary of the deceased Severa G. Andres,
who died May 3, 1951. Payment having been denied by the insurance
company on April 20, 1952, this case was instituted.

ARGUMENTS FOR THE PETITIONER


ISSUE: Whether petitioner insurance company has a cause of action
against private respondent
RULING OF THE COURT

Yes. Article 2207 of the Civil Code is founded on the well-settled principle
of subrogation. If the insured property is destroyed or damaged through
the fault or negligence of a party other than the assured, then the insurer,
upon payment to the assured, will be subrogated to the rights of the
assured to recover from the wrongdoer to the extent that the insurer has
been obligated to pay. Payment by the insurer to the assured operates
as an equitable assignment to the former of all remedies which the latter
may have against the third party whose negligence or wrongful act
caused the loss. The right of subrogation is not dependent upon, nor
does it grow out of, any privity of contract or upon written assignment of
claim. It accrues simply upon payment of the insurance claim by the
insurer [Compania Maritima v. Insurance Company of North America,

Petitioner contends that although the policy has lapsed, the company
waived its right to collect the payment of the premiums as communicated
by the latter in a letter;
If you cannot pay the full amount immediately, send as large an amount
as possible and advise us how soon you expect to be able to pay the
balance. Every consideration will be given to your request consistent
with the company's regulations (Exhibit 4).
If you are unable to cover this amount in full, send us as big an amount
as you are able and we will work out an adjustment most beneficial to
you
Furthermore petitioner paid a partial amount of the balance due

ARGUMENTS FOR THE RESPONDENTS

Respondent contends that since the policy has lapsed, the insured is no
longer protected by the policy the Company did not consider the partial
payment as sufficient consideration for the reinstatement. Petitioners
failure to remit the balance before the death of his wife operated to
deprive him of any right to waive the policy and recover the face value
thereof.
ISSUE
Whether there is a perfected contract of reinstatement after the policy
lapsed due to non-payment of premiums?
RULING OF THE COURT
No. in the case of James McGuire vs. The Manufacturer's Life Insurance
Co. (87 Phil,. 370, 48 Off. Gaz. [1], 114), it was held that The stipulation
in a life insurance policy giving the insured the privilege to reinstate it
upon written application does not give the insured absolute right to such
reinstatement by the mere filing of an application. The Company has the
right to deny the reinstatement if it is not satisfied as to the insurability of
the insured and if the latter does no pay all overdue premium and all
other indebtedness to the Company. After the death of the insured the
insurance Company cannot be compelled to entertain an application for
reinstatement of the policy because the conditions precedent to
reinstatement can no longer be determined and satisfied

DISCUSSION
The supreme court explained that the conditions set forth in
the policy for reinstatement are the following: (a) application shall be
made within three years from the date of lapse; (b) there should be a
production of evidence of the good health of the insured: (c) if the rate of
premium depends upon the age of the Beneficiary, there should likewise
be a production of evidence of his or her good health; (d) there should be
presented such other evidence of insurability at the date of application
for reinstatement; (e) there should be no change which has taken place
in such good health and insurability subsequent to the date of such
application and before the policy is reinstated; and (f) all overdue
premiums and other indebtedness in respect of the policy, together with
interest at six per cent, compounded annually, should first be paid.
The plaintiff-appellant did not comply with the last condition; for he only
paid P100 (on account of the overdue semi-annual premium of P165.15)
on February 20, 1951, before his wife's death (Stipulation, par. 7) ; and,
despite the Company's reminders on April 14 and 27, he remitted the
balance of P65 on May 5, 1951 (received by the Company's agency on
May 11), two days after his wife died. On the face of such facts, the
Company had the right to treat the contract as lapsed and refuse
payment of the policy.

ARGUMENTS: Petitioner insists that Steamship Mutual is a P & I Club is


engaged in the insurance business. To buttress its assertion, it sites the
definition of a P & I club in Hyopsung Maritime Co.,Ltd. vs CA as an
association composed of ship owners in generalwho band together for the
specific purposes of providing insurance cover on a mutual basis against
liabilities incidental to ship owning that the members incur in favor of third
parties. It stresses that as a P & I Club, Steamship Mutuals primary
purpose is to solicit and provide protection and indemnity coverage and for
this purpose, it has engaged the services pioneer to act as its agent.

Respondent contends that although Steamship Mutual is a


P & I Club, it is not engaged in the insurance business in the Philippines. It is
merely as association of vessel owners who have come together to provide
mutual protection against liabilities incidental to ship owning.

ISSUE: Whether or not Steamship Mutual, a Protection and Insurance Club,


engaged in the business in the Philippines.

HELD: The test to determine if a contract is an insurance contract or not,


depends on the nature of the promise, the act required to be performed, and
the exact nature of the agreement in the light of the occurrence, contingency,
or circumstances under which the performance becomes requisite. It is not
by what it is called. Basically, an insurance contract is a contract of
indemnity. In it, one undertakes for a consideration to indemnify another
against loss, damage or liability arising from unknown or contingent event. In
particular, a marine insurance undertakes to indemnify the assured against
marine losses, such as the losses incident to a marine adventure. Section 99
of the Insurance Code enumerates the coverage of marine insurance. By
definition then, Steamship Mutual as a P and I Club is a mutual insurance
association engaged in the marine insurance business.

DISCUSSION ON HOW THE SC RULED THE CASE


A mutual insurance company is a cooperative enterprise where the members
are both the insurer nd insured. the members all contribute, by a system of
premiums or assessments, to the creation of fund from which all losses and
liabilities are paid, and where the profits are divided among themselves, in
proportion to their interest. Additionally, mutual insurance associations, or
clubs, provides three types of coverage, namely, protection and indemnity,

WHITE GOLD MARINE SERVICES ,INC. VS. PIONEER INSURANCEAND

war risk, and defense costs.

SURETY CORPORATION AND THE STEAMSHIP MUTUAL


UNDERWRITING ASSOCIATION
G.R. No. L-44059, 28 October 1977

GULF RESORT,INC
CORPORATION

VS

PHILIPPINE

CHARTER

INSURANCE

G.R. No. 156167 May 16, 2005


FACTS:
White Gold Marine Services, Inc. procured a protection and
indemnity coverage for its vessels from the Steamship Mutual Underwriting
Association Limited through Pioneer Insurance and Surety Corporation.
White Gold was issued a certificate of entry and acceptance. Pioneer also
issued receipts evidencing payments for the coverage. When White Gold
failed to fully pay its accounts, Steamship Mutual refused to renew the
coverage. Steamship Mutual thereafter filed case against White Gold for
collection of sum of money to recover the latters unpaid balance. White Gold
on the other hand, filed a complaint before the Insurance Commission
claiming that Steamship Mutual Violated Sections 186 and 187, while
Pioneer violated Sections 299 to 301 of the Insurance Code.

FACTS :

Gulf Resorts is the owner of the Plaza Resort situated at Agoo, La Union
and had its properties in said resort insured originally with the American
Home Assurance Company (AHAC). In the first 4 policies issued, the
risks of loss from earthquake shock was extended only to petitioners
two swimming pools. Gulf Resorts agreed to insure with Phil Charter the

properties covered by the AHAC policy provided that the policy wording

representative Manuel C. Quijano, categorically stated that its previous


policy, from which respondents policy was copied, covered only
earthquake shock for the two swimming pools.

and rates in said policy be copied in the policy to be issued by Phil


Charter. Phil Charter issued Policy No. 31944 to Gulf Resorts covering

ISSUE: Whether or not the policy covers only the two swimming

the period of March 14, 1990 to March 14, 1991 for P10,700,600.00 for a

pools owned by Gulf Resorts and does not extend to all properties

total premium of P45,159.92. the break-down of premiums shows that

damaged therein

Gulf Resorts paid only P393.00 as premium against earthquake shock


(ES). In Policy No. 31944 issued by defendant, the shock endorsement
provided that In consideration of the payment by the insured to the

HELD: Yes, All the provisions and riders taken and interpreted together,

company of the sum included additional premium the Company agrees,

indubitably show the intention of the parties to extend earthquake shock

notwithstanding what is stated in the printed conditions of this policy due

coverage to the two swimming pools only. Aninsurance premium is the

to the contrary, that this insurance covers loss or damage to shock to any

consideration paid an insurer for undertaking to indemnify the insured

of the property insured by this Policy occasioned by or through or in

against a specified peril. In fire, casualty and marine insurance, the

consequence of earthquake. In Exhibit "7-C" the word "included" above

premium becomes a debt as soon as the risk attaches. In the subject

the underlined portion was deleted. On July 16, 1990 an earthquake

policy, no premium payments were made with regard to earthquake

struck Central Luzon and Northern Luzon and plaintiffs properties

shock coverage except on the two swimming pools. There is no mention

covered by Policy No. 31944 issued by defendant, including the

of any premium payable for the other resort properties with regard to

two swimming pools in its Agoo Playa Resort were damaged.

earthquake

shock.

This

is

consistent

with

the

history

of

petitionersinsurance policies with AHAC.

Petitioner advised respondent that it would be making a claim under its


Insurance Policy 31944 for damages on its properties. Respondent
denied petitioners claim on the ground that its insurance policy only
afforded earthquake shock coverage to the twoswimming pools of the
resort. The trial court ruled in favor of respondent. In its ruling,
the schedule clearly shows that petitioner paid only a premium of

DISCUSSION ON HOW THE SC RULED THE CASE


A careful examination of the premium recapitulation will show that it is
the clear intent of the parties to extend earthquake shock coverage only
to the two swimming pools. Section 2(1) of the Insurance Code defines a
contract of insurance as an agreement whereby one undertakes for a
consideration to indemnify another against loss, damage or liability
arising from an unknown or contingent event. Thus, an insurance
contract exists where the following elements concur:

P393.00 against the peril of earthquake shock, the same premium it had
1. The insured has an insurable interest;
paid against earthquake shock only on the two swimming pools in all the
policies issued by AHAC.

2. The insured is subject to a risk of loss by the happening of


the designated peril;

ARGUMENTS:

3. The insurer assumes the risk;

Petitioner contends:

4. Such assumption of risk is part of a general scheme to


distribute actual losses among a large group of persons
bearing a similar risk; and

First, that the policys earthquake shock endorsement clearly covers all
of the properties insured and not only the swimming pools. It used the
words "any property insured by this policy," and it should be interpreted
as all inclusive.
Respondent Arguments:
None of the previous policies issued by AHAC-AIU from 1983 to 1990
explicitly extended coverage against earthquake shock to petitioners
insured properties other than on the two swimming pools. Petitioner
admitted that from 1984 to 1988, only the two swimming pools were
insured against earthquake shock. From 1988 until 1990, the provisions
in its policy were practically identical to its earlier policies, and there was
no increase in the premium paid. AHAC-AIU, in a letter 19 by its

5. In consideration of the insurer's promise, the insured


pays a premium.
An insurance premium is the consideration paid an insurer for
undertaking to indemnify the insured against a specified peril. 27 In fire,
casualty, and marine insurance, the premium payable becomes a debt
as soon as the risk attaches.28 In the subject policy, no premium
payments were made with regard to earthquake shock coverage, except
on the two swimming pools. There is no mention of any premium payable
for the other resort properties with regard to earthquake shock. This is
consistent with the history of petitioners previous insurance policies from
AHAC-AIU. As borne out by petitioners witnesses.

SUMMIT GUARANTY & INSURANCE COMPANY, INC., vs.THE


HONORABLE GREGORIA C. ARNALDO, in her capacity as
Insurance Commissioner, and FGU INSURANCE CORPORATION
G.R. No. L-48546 February 29, 1988

FACTS:
As a result of a vehicular accident that happened on November 26,1976
whereby a Ford Pick-up with Plate No. UC-5925 Phil. '76 owned by
Marcos Olaso was bumped by a cargo truck with Plate No. OY-783 then
owned by Alberto Floralde, FGU insurance poration FGU by reason of
Motor Vehicle Insurance Policy No. IC-VF-07185 paid Olaso the sum of
P 2,817.50 as its share in the repair cost of the said Ford Pick-up.
Having thus been subrogated to the rights and causes of action of said
Olaso in the said amount FGU formally demanded payment of said
amount from Floralde and attempted to verify Floralde's insurance
carrier. Floralde failed to reveal his insurance carrier. In the early part of
1978 FGU was able to ascertain the Identity of Floralde's insurance
carrier to be the Summit Guaranty and Insurance Company, Inc.
(Summit). On February 22,1978 FGU wrote to the insurance
commissioner requesting for a conference with Summit and demanded
from Summit through counsel on February 28,1978 the payment of the
damages sustained by the car of Olaso but to no avail. Hence on May
22,1978 FGU filed IC Case No. 825 in the Insurance Commissioner's
Office against Summit for recovery of said amount.
ARGUMENTS:
Petitioner squarely brings into focus the provisions of Section 384 of PD
612, the Insurance Code, Petitioner company contends that the two
periods prescribed in the aforementioned law that is, the six-month
period for filing the notice of claim and the one-year period for bringing
an action or suit are mandatory and must always concur.
Respondent FGU, however, contends that the said one-year prescriptive
period can not apply to it because it was merely subrogated to the rights
of Olaso. Respondent Commissioner invites attention to the phrase "in
proper cases" in Section 384 of PD 612 and argues that the prescriptive
period was interrupted upon the extrajudicial demand for payment made
by FGU on petitioner
ISSUE:
Whether the claim of Insurance Corporation is barred by prescription
under section 384 of PD No. 612.
HELD:
In the present case, it is not denied that an extrajudicial demand for
payment was made by respondent FGU on petitioner but petitioner failed
to respond to the same. Nevertheless the complaint was filed even
before a denial of the claim was made by petitioner. For all legal
purposes, the one-year prescriptive period provided for in Section 384 of
the Insurance Code has not begun to run. The cause of action arises
only and starts to run upon the denial of the claim by the insurance
company.

November 15, 2001

FACTS:
The facts show that Caltex Philippines (Caltex for brevity) entered into a
contract of affreightment with the petitioner, Delsan Transport Lines, Inc.,
for a period of one year whereby the said common carrier agreed to
transport Caltexs industrial fuel oil from the Batangas-Bataan Refinery to
different parts of the country. Under the contract, petitioner took on board
its vessel, MT Maysun 2,277.314 kiloliters of industrial fuel oil of Caltex
to be delivered to the Caltex Oil Terminal in Zamboanga City. The
shipment was insured with the private respondent, American Home
Assurance Corporation.
On August 14, 1986, MT Maysum set sail from Batangas for Zamboanga
City. Unfortunately, the vessel sank in the early morning of August 16,
1986 near Panay Gulf in the Visayas taking with it the entire cargo of fuel
oil.
Subsequently, private respondent paid Caltex the sum of Five Million
Ninety-Six Thousand Six Hundred Thirty-Five Pesos and Fifty-Seven
Centavos (P5,096,635.67) representing the insured value of the lost
cargo. Exercising its right of subrogation under Article 2207 of the New
Civil Code, the private respondent demanded of the petitioner the same
amount it paid to Caltex.1wphi1.nt
Due to its failure to collect from the petitioner despite prior demand,
private respondent filed a complaint with the Regional Trial Court of
Makati City, Branch 137, for collection of a sum of money.
ARGUMENTS:
Petitioner Delsan Transport Lines, Inc. invokes the provision of Section
113 of the Insurance Code of the Philippines, which states that in every
marine insurance upon a ship or freight, or freightage, or upon any thin
which is the subject of marine insurance there is an implied warranty by
the shipper that the ship is seaworthy.
Private respondent argues that the vessel was not seaworthy
ISSUE: Whether or not the payment made by the private respondent to
Caltex for the insured value of the lost cargo amounted to an admission
that the vessel was seaworthy, thus precluding any action for recovery
against the petitioner.
HELD:
The payment made by the private respondent for the insured value of the
lost cargo operates as waiver of its (private respondent) right to enforce
the term of the implied warranty against Caltex under the marine
insurance policy. However, the same cannot be validly interpreted as an
automatic admission of the vessels seaworthiness by the private
respondent as to foreclose recourse against the petitioner for any liability
under its contractual obligation as a common carrier. The fact of payment
grants the private respondent subrogatory right which enables it to
exercise legal remedies that would otherwise be available to Caltex as
owner of the lost cargo against the petitioner common carrier.
DISCUSSION ON HOW THE SC RULED THE CASE

DISCUSSION ON HOW THE SC RULED THE CASE


We find no merit in the contention of petitioner company. There is
absolutely nothing in the law which mandates that the two periods must
always concur. On the contrary, it is very clear that the one-year period is
only required 'in proper cases.' It appears that petitioner company
disregarded this very significant phrase when it made its own
interpretation of the law. Had the lawmakers intended it to be the way
petitioner company assumes it to be, then the phrase in proper cases'
would not have been inserted.
DELSAN
TRANSPORT
LINES,
vs.
THE HON. COURT OF APPEALS and
ASSURANCE CORPORATION, respondents.

G.R. No. 127897

INC., petitioner,
AMERICAN

HOME

The right of subrogation has its roots in equity. It is designed to promote


and to accomplish justice and is the mode which equity adopts to compel
the ultimate payment of a debt by one who in justice and good
conscience ought to pay.9 It is not dependent upon, nor does it grow out
of, any privity of contract or upon written assignment of claim. It accrues
simply upon payment by the insurance company of the insurance
claim.10 Consequently, the payment made by the private respondent
(insurer) to Caltex (assured) operates as an equitable assignment to the
former of all the remedies which the latter may have against the
petitioner.

PERLA COMPANIA DE SEGUROS, INC., PETITIONER, V. HON.


CONSTANTE A. ANCHETA., RESPONDENTS. GR L-49699. AUGUST 8,
1988
Case no. 43
Facts: IH Scout in which private respondents were riding collided with
Superlines bus, private respondent sustained physical injuries in varying
degrees of gravity. Thus, they filed a complaint for damages against
Superlines, the bus driver and petitioner, the insurer of the bus.
The
judge ordered petitioner to pay private respondents the 'no fault
indemnity in the amount of P5,000. Petitioner contended that under Sec.
378 of the Insurance Code, the insurer liable to pay the P5,000.00 is the
insurer of the vehicle in which private respondents were riding, not
petitioner. Motions for Reconsiderations were denied.
Issue: whether or not petitioner is the insurer liable to indemnify private
respondents under Sec. 378 of the Insurance Code.
Ruling: under the "no fault indemnity" provision, where proof of fault or
negligence is not necessary for payment of any claim for death Or injury
to a passenger or a third party, are established:
1. A claim may be made against one motor vehicle only.
2. If the victim is an occupant of a vehicle, the claim shall lie against the
insurer of the vehicle. in which he is riding, mounting or dismounting
from.
3. In any other case (i.e. if the victim is not an occupant of a vehicle), the
claim shall lie against the insurer of the directly offending vehicle.
4. In all cases, the right of the party paying the claim to recover against
the owner of the vehicle responsible for the accident shall be maintained.
The claimant is not free to choose from which insurer he will claim the
"no fault indemnity," as the law, by using the word "shall, makes it
mandatory that the claim be made against the insurer of the vehicle in
which the occupant is riding, mounting or dismounting from.
That said vehicle might not be the one that caused the accident is of no
moment since the law itself provides that the party paying the claim
under Sec. 378 may recover against the owner of the vehicle responsible
for the accident. This is precisely the essence of "no fault indemnity"
insurance which was introduced to and made part of our laws in order to
provide victims of vehicular accidents or their heirs immediate
compensation, although in a limited amount, pending final determination
of who is responsible for the accident and liable for the victims'injuries or
death. Irrespective of whether or not fault or negligence lies with the
driver of the Superlines bus, as private respondents were not occupants
of the bus, they cannot claim the "no fault indemnity" provided in Sec.
378 from petitioner. The claim should be made against the insurer of the
vehicle they were riding.

VILLACORTA V. INSURANCE COMMISSION, PETITIONER AND EMPIRE


INSURANCE COMPANY, RESPONDENTS
G.R. NO. L-54171 OCTOBER 28, 1980
Case no. 44
FACTS: PETITIONER VILLACORTA HAD HER COLT LANCER CAR INSURED WITH
EMPIRE INSURANCE COMPANY AGAINST OWN DAMAGE, THEFT AND 3 RD PARTY
LIABILITY. WHILE THE CAR WAS IN THE REPAIR SHOP, ONE OF THE EMPLOYEES
OF THE SAID REPAIR SHOP TOOK IT OUT FOR A JOYRIDE AFTER WHICH IT
FIGURED IN A VEHICULAR ACCIDENT. THIS RESULTED TO THE DEATH OF THE
DRIVER AND SOME OF THE PASSENGERS AS WELL AS TO EXTENSIVE DAMAGE
TO THE CAR. VILLACORTA FILED A CLAIM FOR TOTAL LOSS WITH THE SAID
INSURANCE COMPANY. HOWEVER, IT DENIED THE CLAIM ON THE GROUND
THAT THE ACCIDENT DID NOT FALL WITHIN THE PROVISIONS OF THE POLICY
EITHER FOR THE OWN DAMAGE OR THEFT COVERAGE, INVOKING THE POLICY
PROVISION ON AUTHORIZED DRIVER CLAUSE WHICH STATES THAT THE
POLICY LIMITS THE USE OF THE INSURED VEHICLE TO TWO (2) PERSONS
ONLY, NAMELY: THE INSURED HIMSELF OR ANY PERSON ON HIS (INSURED'S)
PERMISSION. THIS WAS UPHELD BY THE INSURANCE COMMISSION FURTHER
STATING THAT THE CAR WAS NOT STOLEN AND THEREFORE NOT COVERED BY
THE THEFT CLAUSE BECAUSE IT IS NOT EVIDENT THAT THE PERSON WHO
TOOK THE CAR FOR A JOYRIDE INTENDS TO PERMANENTLY DEPRIVE THE
INSURED OF HIS/ HER CAR.

ISSUE: WHETHER OR NOT THE INSURER COMPANY SHOULD PAY THE SAID
CLAIM.
RULING: YES. WHERE THE INSUREDS CAR IS WRONGFULLY TAKEN WITHOUT
THE INSUREDS CONSENT FROM THE CAR SERVICE AND REPAIR SHOP TO
WHOM IT HAD BEEN ENTRUSTED FOR CHECK-UP AND REPAIRS (ASSUMING
THAT SUCH TAKING WAS FOR A JOY RIDE, IN THE COURSE OF WHICH IT WAS
TOTALLY SMASHED IN AN ACCIDENT), RESPONDENT INSURER IS LIABLE AND
MUST PAY INSURED FOR THE TOTAL LOSS OF THE INSURED VEHICLE UNDER
THE THEFT CLAUSE OF THE POLICY. ASSUMING, DESPITE THE TOTALLY
INADEQUATE EVIDENCE, THAT THE TAKING WAS TEMPORARY AND FOR A JOY
RIDE, THE COURT SUSTAINS AS THE BETTER VIEW THAT WHICH HOLDS THAT
WHEN A PERSON, EITHER WITH THE OBJECT OF GOING TO A CERTAIN PLACE,
OR LEARNING HOW TO DRIVE, OR ENJOYING A FREE RIDE, TAKES POSSESSION
OF A VEHICLE BELONGING TO ANOTHER, WITHOUT THE CONSENT OF ITS
OWNER, HE IS GUILTY OF THEFT BECAUSE BY TAKING POSSESSION OF THE
PERSONAL PROPERTY BELONGING TO ANOTHER AND USING IT, HIS INTENT TO
GAIN IS EVIDENT SINCE HE DERIVES THERE FROM UTILITY, SATISFACTION,
ENJOYMENT AND PLEASURE. ACCORDINGLY, THE APPEALED DECISION IS
SET ASIDE AND JUDGMENT IS HEREBY RENDERED SENTENCING PRIVATE
RESPONDENT TO PAY PETITIONER THE SUM OF P35,000.00 WITH LEGAL
INTEREST FROM THE FILING OF THE COMPLAINT UNTIL FULL PAYMENT IS MADE
AND TO PAY THE COSTS OF SUIT.

HEIRS OF LORETO C. MARAMAG, PETITIONERS V. EVA VERNA DE


GUZMAN ET AL., RESPONDENTS G R 181132. JULY 5, 2009
Case no. 45
Facts: Petitioners filed a writ of preliminary injunction alleging that they
were the legitimate wife and children of Loreto Maramag (Loreto), while
respondents were Loretos illegitimate family; Eva de Guzman Maramag
(Eva) was a concubine of Loreto and a suspect in the killing of the latter,
thus she is disqualified to receive any proceeds from his insurance
policies and the illegitimate children of Loreto Odessa, Karl, Brian and
Trisha Angelie were entitled only to one-half of the legitime children, thus
petitioners could not be deprived of their legitime. The insurance
company claimed that it was bound to honor the insurance policies
designating the children of Loreto with Eva as beneficiaries pursuant to
Section 53 of the Insurance and not to the estate or to the heirs of the
insured. The RTC granted the petition which states that civil code does
not apply. It is very clear under Sec. 53 thereof that the insurance
proceeds shall be applied exclusively to the proper interest of the person
in whose name or for whose benefit it is made, unless otherwise
specified in the policy. The CA dismissed the case for lack of jurisdiction
for filing beyond reglementary period.
Issue: W/N Eva can claim even though prohibited under the civil code
against donation.
Ruling: YES. Petition is DENIED.
Any person who is forbidden from receiving any donation under Article
739 cannot be named beneficiary of a life insurance policy of the person
who cannot make any donation to him. If a concubine is made the
beneficiary, it is believed that the insurance contract will still remain valid,
but the indemnity must go to the legal heirs and not to the concubine, for
evidently, what is prohibited under Art. 2012 is the naming of the
improper beneficiary.
SECTION 53. The insurance proceeds shall be applied exclusively to the
proper interest of the person in whose name or for whose benefit it is
made unless otherwise specified in the policy.Only persons entitled to
claim the insurance proceeds are either the insured, if still alive; or the
beneficiary, if the insured is already deceased, upon the maturation of
the policy.
The revocation of Eva as a beneficiary in one policy and her
disqualification as such in another are of no moment considering that the
designation of the illegitimate children as beneficiaries in Loretos
insurance policies remains valid. Because no legal proscription exists in
naming as beneficiaries the children of illicit relationships by the insured,
the shares of Eva in the insurance proceeds, whether forfeited by the
court in view of the prohibition on donations under Article 739 of the Civil
Code or by the insurers themselves for reasons based on the insurance

contracts, must be awarded to the said illegitimate children, the


designated beneficiaries, to the exclusion of petitioners. It is only in
cases where the insured has not designated any beneficiary, or when the
designated beneficiary is disqualified by law to receive the proceeds, that
the insurance policy proceeds shall redound to the benefit of the estate
of the insured.
ETERNAL GARDENS VS PHILIPPINE AMERICAN LIFE INSURANCE
G.R. NO. 166245 APRIL 9, 2008
FACTS:
RESPONDENT PHILIPPINE AMERICAN LIFE INSURANCE COMPANY
(PHILAMLIFE) ENTERED INTO A CREDITOR GROUP LIFE POLICY WITH
PETITIONER
ETERNAL GARDENS MEMORIAL PARK CORPORATION
(ETERNAL). UNDER THE POLICY, THE CLIENTS OF ETERNAL WHO PURCHASED
BURIAL LOTS FROM IT ON INSTALLMENT BASIS WOULD BE INSURED BY
PHILAMLIFE. ETERNAL WAS REQUIRED UNDER THE POLICY TO SUBMIT TO
PHILAMLIFE A LIST OF ALL NEW LOT PURCHASERS, TOGETHER WITH A COPY
OF THE APPLICATION OF EACH PURCHASER, AND THE AMOUNTS OF THE
RESPECTIVE UNPAID BALANCES OF ALL INSURED LOT PURCHASERS.
ETERNAL COMPLIED BY SUBMITTING A LETTER CONTAINING A LIST OF
INSURABLE BALANCES OF ITS LOT BUYERS, ONE OF THOSE INCLUDED A
CERTAIN JOHN CHUANG WHO SUBSEQUENTLY DIED.

ETERNAL GARDENS COMPLIED WITH THE REQUIREMENTS SET FORTH BY


PHILAMLIFE. HOWEVER, ONE YEAR HAD ALREADY ELAPSED YET PHILAMLIFE
STILL HAS NO REPLY. THIS PROMPTED ETERNAL TO DEMAND FROM
PHILAMLIFE THE BALANCE DUE TO CHUANG.
ARGUMENTS:
PHILAMLIFE DENIES THE CLAIM OF ETERNAL CONTENDING THAT NO
APPLICATION WAS FILED IN THEIR OFFICE PRIOR TO THE DEATH AS THE
POLICY STATES THAT "THERE SHALL BE NO INSURANCE IF THE APPLICATION
IS NOT APPROVED BY THE COMPANY." THUS, THEY ARE NOT LIABLE.

ETERNAL CLAIMS THAT THE EVIDENCE THAT IT PRESENTED BEFORE THE


TRIAL COURT SUPPORTS ITS CONTENTION THAT IT SUBMITTED A COPY OF THE
INSURANCE APPLICATION OF CHUANG BEFORE HIS DEATH.

ISSUE:
WHETHER OR NOT PHILAMLIFE RECEIVED THE APPLICATION PRIOR TO THE
DEATH OF CHUANG AND THEREBY BE HELD LIABLE.

BELIEVE A VEHICLE IS INSURABLE. SONGCO, ON THE OTHER HAND, ASSERTS


THAT THEY SHOULD BE ENTITLED FOR THE PAYMENT .

ISSUE:
WHETHER OR NOT FIELDMENS SHOULD BE HELD LIABLE.
RULING OF THE COURT:
YES.
IT IS A WELL-SETTLED JURISPRUDENCE THAT WHERE INEQUITABLE CONDUCT
IS SHOWN BY AN INSURANCE FIRM, IT IS "ESTOPPED FROM ENFORCING
FORFEITURES IN ITS FAVOR, IN ORDER TO FORESTALL FRAUD OR IMPOSITION
ON THE INSURED." AS ESTOPPEL IS PRIMARILY BASED ON THE DOCTRINE OF
GOOD FAITH AND THE AVOIDANCE OF HARM THAT WILL BEFALL THE
INNOCENT PARTY DUE TO ITS INJURIOUS RELIANCE, THE FAILURE TO APPLY IT
IN THIS CASE WOULD RESULT IN A GROSS TRAVESTY OF JUSTICE.

AS STATED IN QUA CHEE GAN CASE, THE CONTRACT OF INSURANCE IS ONE


OF PERFECT GOOD FAITH (UBERIMA FIDES) NOT FOR THE INSURED ALONE,
BUT EQUALLY SO FOR THE INSURER; IN FACT , IT IS MORE SO FOR THE
LATTER, SINCE ITS DOMINANT BARGAINING POSITION CARRIES WITH IT
STRICTER RESPONSIBILITY. THIS IS MERELY TO STRESS THAT WHILE THE
MORALITY OF THE BUSINESS WORLD IS NOT THE MORALITY OF INSTITUTIONS
OF RECTITUDE LIKE THE PULPIT AND THE ACADEME, IT CANNOT DESCEND SO
LOW AS TO BE ANOTHER NAME FOR GUILE OR DECEPTION. MOREOVER,
SHOULD IT HAPPEN THUS, NO COURT OF JUSTICE SHOULD ALLOW ITSELF TO
LEND ITS APPROVAL AND SUPPORT.

SERVICEWIDE SPECIALIST VS COURT OF APPEALS


G.R. NO. 110597 MAY 8, 1996
FACTS:
PRIVATE RESPONDENT SPOUSES TRINIDAD PURCHASED AN ISUZU GEMINI
CAR ON INSTALLMENTS FROM AUTOWORLD SALES CORPORATION. TO
SECURE PAYMENT, TRINIDADS EXECUTED A PROMISSORY NOTE AND A DEED
OF CHATTEL MORTGAGE IN FAVOUR OF AUTOWORLD. THE LATTER ASSIGNED
ITS INTEREST TO FILINVEST, A FACT KNOWN TO TRINIDADS. THUS, TRINIDADS
ISSUED POSTDATED CHECKS TO FULLY PAY THE CAR IN FAVOUR OF
FILINVEST. FILINVEST ASSIGNED ALL ITS RIGHTS IN FAVOUR OF SERVICEWIDE
WHO DEMANDED PAYMENT FROM TRINIDADS. WHEN THE LATTER REFUSED TO
PAY, SERVICEWIDE FILED WITH THE MUNICIPAL TRIAL COURT AN ACTION FOR
REPLEVIN AND DAMAGES.

RULING OF THE COURT:


YES.
INSURANCE CONTRACTS ARE WHOLLY PREPARED BY THE INSURER WITH VAST

MTC RULED IN FAVOR OF SERVICEWIDE. RTC REVERSED THE DECISION. CA


AFFIRMED THE DECISION OF THE RTC. HENCE, THIS PETITION.

AMOUNTS OF EXPERIENCE IN THE INDUSTRY PURPOSEFULLY USED TO ITS


ADVANTAGE. MORE OFTEN THAN NOT, INSURANCE CONTRACTS ARE
CONTRACTS OF ADHESION CONTAINING TECHNICAL TERMS AND CONDITIONS
OF THE INDUSTRY, CONFUSING IF AT ALL UNDERSTANDABLE TO LAYPERSONS,
THAT ARE IMPOSED ON THOSE WHO WISH TO AVAIL OF INSURANCE. AS SUCH,
INSURANCE CONTRACTS ARE IMBUED WITH PUBLIC INTEREST THAT MUST BE
CONSIDERED WHENEVER THE RIGHTS AND OBLIGATIONS OF THE INSURER
AND THE INSURED ARE TO BE DELINEATED. HENCE, IN ORDER TO PROTECT
THE INTEREST OF INSURANCE APPLICANTS, INSURANCE COMPANIES MUST BE
OBLIGATED TO ACT WITH HASTE UPON INSURANCE APPLICATIONS, TO EITHER
DENY OR APPROVE THE SAME, OR OTHERWISE BE BOUND TO HONOR THE
APPLICATION AS A VALID, BINDING, AND EFFECTIVE INSURANCE CONTRACT .

ARGUMENTS:
PETITIONER CONTENDS THAT THE MATTER ABOUT THE NOTICE IS DEEMED

THUS, THE MERE INACTION OF THE INSURER DOES NOT TERMINATE THE
CONTRACT. HAVING PROVED THAT PHILAMLIFE RECEIVED THE LETTER PRIOR
TO THE DEATH AND FAILURE ON THE PART OF THE LATTER TO PRODUCE
EVIDENCE AGAINST IT, PHILAMLIFE IS DEEMED TO HAVE RECEIVED THE
APPLICATION.
FIELDMENS INSURANCE CO. VS. VDA DE SONGCO
G.R. NO. 24833 SEPTEMBER 23, 1986

FACTS:
FEDERICO SONGCO, OWNER OF A PRIVATE JEEPNEY, WAS INDUCED BY THE
AGENT OF FIELDMENS INSURANCE CO. TO APPLY FOR A COMMON
CARRIERS INSURANCE POLICY WHICH WAS LATER RENEWED. DURING THE
EFFECTIVITY OF THE RENEWED POLICY, THE JEEPNEY COLLIDED WITH A CAR
RESULTING DEATH TO FEDERICO AND HIS SON. NOW, THE SURVIVING SPOUSE
OF FEDERICO SEEKS RELIEF TO FIELDMENS. THE COMPANY REFUSED TO
PAY.
ARGUMENTS:
THE AGENT OF FIELDMENS CONTENDS THAT SINCE THEY ARE NOT
GOVERNMENT-OWNED, THEY COULD DO WHAT THEY PLEASE WHENEVER THEY

WAIVED BY PRIVATE RESPONDENTS BECAUSE THE CAR SHOULD BE FULLY


COVERED AT ALL TIMES. PETITIONER CLAIMS THAT IF, AS STATED IN THE
CHATTEL MORTGAGE, PRIVATE RESPONDENTS FAILED TO RENEW THE
INSURANCE, PETITIONER IS ENTITLED TO RENEW THE SAME FOR THE
ACCOUNT OF PRIVATE RESPONDENTS WITHOUT ANY NOTICE TO THEM.

ISSUE:
WHETHER OR NOT PETITIONER SHOULD HAVE APPLIED THE INSTALLMENT
PAYMENTS MADE BY PRIVATE RESPONDENTS FOR THE PAYMENT OF THE CAR
TO THE PAYMENT OF THE INSURANCE PREMIUMS WITHOUT PRIOR NOTICE TO
PRIVATE RESPONDENTS.

RULING OF THE COURT:


NO.
WHILE IT IS TRUE THAT THE CHATTEL MORTGAGE DOES NOT SAY THAT
NOTICE TO THE MORTGAGOR OF THE RENEWAL OF THE INSURANCE PREMIUM
BY THE MORTGAGEE IS NECESSARY, AT THE SAME TIME, THERE IS NO
PROVISION THAT AUTHORIZES PETITIONER TO APPLY THE PAYMENTS MADE TO
IT FOR THE PAYMENT OF THE CHATTEL TO THE PAYMENT OF THE SAID
PREMIUMS. FROM THE RECORDS OF THE CASE, IT IS CLEAR THAT PRIVATE
RESPONDENTS HAD FULLY PAID FOR THE CAR. THIS FACT WAS NEVER
REBUTTED BY PETITIONER. PETITIONER AS MORTGAGEE WAS NOT DUTYBOUND TO RENEW THE INSURANCE IN THE EVENT THAT PRIVATE
RESPONDENTS FAILED TO DO SO; IT WAS MERELY OPTIONAL ON ITS PART .

AS TO THE ATTORNEYS FEES, ARTICLE 2208 OF THE CIVIL CODE ALLOWS


ATTORNEY'S FEES TO BE AWARDED BY A COURT WHEN ITS CLAIMANT IS
COMPELLED TO LITIGATE WITH THIRD PERSONS OR TO INCUR EXPENSES TO
PROTECT HIS INTEREST BY REASON OF AN UNJUSTIFIED ACT OR OMISSION ON
THE PART OF THE PARTY FROM WHOM IT IS SOUGHT. TO BE SURE, PRIVATE
RESPONDENTS WERE FORCED TO LITIGATE TO PROTECT THEIR RIGHTS BUT
AS WE HAVE PREVIOUSLY HELD: "WHERE NO SUFFICIENT SHOWING OF BAD

FAITH WOULD BE REFLECTED IN A PARTY'S PERSISTENCE IN A CASE OTHER


THAN AN ERRONEOUS CONVICTION OF THE RIGHTEOUSNESS OF HIS CAUSE,
ATTORNEY'S FEE SHALL NOT BE RECOVERED AS COST."

ASIAN TERMINALS, INC., VS. MALAYAN INSURANCE CO.


G.R. NO. 171406 APRIL 4, 2011
FACTS:
THE SHIPMENT OF SHANDONG WEIFANG SODA ASH PLANT WAS INSURED
WITH RESPONDENT MALAYAN INSURANCE COMPANY (MALAYAN). DUE TO THE
UNLOADING DONE BY THE STEVEDORES OF PETITIONER ASIAN TERMINALS,
INC., (ATI), A TOTAL OF 2,881 BAGS WERE IN BAD ORDER CONDITION DUE TO
SPILLAGE, CAKING, AND HARDENING OF THE CONTENTS.
MALAYAN, AS INSURER, PAID THE VALUE OF THE LOST/ DAMAGED CARGOES
TO THE CONSIGNEE HOWEVER ASKS FOR REIMBURSEMENT TO ATI.
ARGUMENTS:
ATI CONTENDS THAT RESPONDENT HAS NO CAUSE OF ACTION BECAUSE IT
FAILED TO PRESENT THE INSURANCE CONTRACT OR POLICY COVERING THE
SUBJECT SHIPMENT. THUS, PETITIONER SUBMITS THAT WITHOUT PROOF OF A
VALID
SUBROGATION,
RESPONDENT
IS
NOT
ENTITLED
TO
ANY
REIMBURSEMENT.

MALAYAN, ON THE OTHER HAND, ARGUES THAT UNDER PREVAILING


JURISPRUDENCE, PRESENTATION OF THE INSURANCE POLICY IS NOT
INDISPENSABLE. RESPONDENT FURTHER AVERS THAT "THE RIGHT OF
SUBROGATION HAS ITS ROOTS IN EQUITY - IT IS DESIGNED TO PROMOTE AND
TO ACCOMPLISH JUSTICE AND IS THE MODE WHICH EQUITY ADOPTS TO
COMPEL THE ULTIMATE PAYMENT OF A DEBT BY ONE WHO IN JUSTICE, EQUITY
AND GOOD CONSCIENCE OUGHT TO PAY."

ISSUE:
WHETHER OR NOT MALAYAN HAS A RIGHT OF RELIEF AGAINST ATI.
RULING:
YES.
Non-presentation of the insurance contract or policy is not necessarily
fatal. The subrogation receipt, by itself, is sufficient to establish not only
the relationship of insurer and the assured shipper of the lost cargo, but
also the amount paid to settle the insurance claim. The right of
subrogation accrues simply upon payment by the insurance company of
the insurance claim.
Since there was no issue regarding the validity of the insurance contract
or policy, or any provision thereof, respondent had no reason to present
the insurance contract or policy as evidence during the trial.
THE
MANUFACTURERS
LIFE
INSURANCE
CO
vs.
BIBIANO L. MEER, in the capacity as Collector of Internal Revenue

Manufacturer contended that when it made premium loans or


premium advances by virtue of the non-forfeiture clauses, it did not
collect premiums within the meaning of the above sections of the law,
and therefore it is not amendable to the tax provided.

ISSUE/S:
(a) Whether or not premium advances made by plaintiffappellant under the automatic premium loan clause of its policies are
"premium collected" by the Company subject to tax
(b) Whether or not, in the application of the automatic premium
loan clause of plaintiff-appellant's policies, there is "payment in money,
notes, credit, or any substitutes for money

RULING OF THE COURT:


BOTH IN AFFIRMATIVE.
RATIONALE OF THE COURT BY CITING AN INSTANCE:
1. A person secures a 20-years endowment policy for P5,000 from
Manufacturers and pays an annual premium of P250. He pays the first
ten yearly premiums amounting to P2,500 and on this amount plaintiffappellant pays the taxes. Also, the cash value of said policy after the
payment of the 10th annual premium amounts to P1,000." When on the
eleventh year the annual premium fell due and the insured remitted no
money within the grace period, the insurer treated the premium then
overdue as paid from the cash value, the amount being loan to the
policyholder who could discharge it at anytime with interest at 6 per cent.
The insurance contract, therefore, continued in force for the eleventh
year.
Under the circumstances described, did the insurer collect the amount of
P250 as the annual premium for the eleventh year on the said policy? In
effect the Manufacturers Life Insurance Co. loaned to the person P250
and the latter in turn paid with that sum the annual premium on his policy.
The Company therefore collected the premium for the eleventh year.
"How could there be such a collection when insurer becomes a creditor,

FACTS:
Manufacturer Life Insurance Company is a corporation duly
organized in Canada with head office at Toronto. It is duly registered and
licensed to engage in life insurance business in the Philippines, and
maintains a branch office in Manila. It was engaged in such business in
the Philippines for more than five years before and including the year
1941. But due to the exigencies of the war it closed the branch office at
Manila during 1942 up to September 1945.
In the course of its operations before the war, plaintiff issued a
number of life insurance policies in the Philippines containing stipulations
referred to as non-forfeiture clauses.
Since the insured failed to pay from 1942 to 1946, the
company applied the provision of the automatic premium loan clauses;
and the net amount of premiums so advanced or loaned totaled
P1,069,254.98. On this sum the defendant Collector of Internal Revenue
assessed P17,917.12. The assessment was made pursuant to section
255 of the NIRC which put taxes on insurance premiums paid by money,
notes, credits or any substitutes for money.

acquires a lien on the policy and is entitled to collect interest on the


amount of the unpaid premiums?".
Wittingly, the "premium" and the "loan" have been interchanged in the
argument. The insurer "became a creditor" of the loan, but not of the
premium that had already been paid. And it is entitled to collect interest
on the loan, not on the premium.
The insured paid the premium for the eleventh; but in turn he became a
debtor of the company for the sum of P250. This debt he could repay
either by later remitting the money to the insurer or by letting the cash
value compensate for it. The debt may also be deducted from the
amount of the policy should he die thereafter during the continuance of
the policy.

There was new credit for the advances made. True, the company could
not sue the insured to enforce that credit. But it has means of satisfaction

Whether a warranty referred to in the policy as forming part of


the contract of insurance and in the form of a rider to the insurance
policy, is null and void because not complying with the Philippine
Insurance Act

out of the cash surrender value.


RULING OF THE COURT
Here again it may be urged that if the credit is paid out of the cash
surrender value, there were no new funds added to the company's

The Insurance Act, Section 65, taken from California law,


states:

assets. Cash surrender value "as applied to life insurance policy, is the
amount of money the company agrees to pay to the holder of the policy if
he surrenders it and releases his claims upon it. The more premiums the
insured has paid the greater will be the surrender value; but the

"Every express warranty, made at or before the execution of a policy,


must be contained in the policy itself, or in another instrument signed by
the insured and referred to in the policy, as making a part of it."

surrender value is always a lesser sum than the total amount of


premiums paid."

Warranty F, indemnifying for a value of Php 20,000 and pasted


on the left margin of the policy stated:

The cash value or cash surrender value is therefore an amount which


the insurance company holds in trust for the insured to be delivered to
him upon demand. It is therefore a liability of the company to the insured.
Now then, when the company's credit for advances is paid out of the
cash value or cash surrender value, that value and the company's
liability is thereby dismissed. Consequently, the net assets of the
insurance company increase.

It is hereby declared and agreed that during the currency of


this policy no hazardous goods be stored in the Building to which this
insurance applies or in any building communicating therewith, provided,
always, however, that the Insured be permitted to stored a small quantity
of the hazardous goods specified below, but not exceeding in all 3 per
cent of the total value of the whole of the goods or merchandise
contained in said warehouse, viz; . . . .
Also, the court stated a book that said, "any express warranty or

2.

The insurer agreed to consider the premium paid on the


strength of the automatic loan. The premium was paid by
means of a "note" or "credit" or "other substitute for money"
and the taxes due because section 255 above quoted levies

condition is always a part of the policy, but, like any other part of an
express contract, may be written in the margin, or contained in proposals
or documents expressly referred to in the policy, and so made a part of
it."

taxes according to the total premiums collected by the insurer


"whether such premiums are paid in money, notes, credits or
any substitutes for money.

It is well settled that a rider attached to a policy is a part of the


contract, to the same extent and with like effect as it actually embodied
therein. In the second place, it is equally well settled that an express
warranty must appear upon the face of the policy, or be clearly

ANG GIOK CHIP,


vs.
SPRINGFIELD FIRE & MARINE INSURANCE COMPANY

incorporated therein and made a part thereof by explicit reference, or by


words clearly evidencing such intention.
The court concluded that Warranty F is contained in the policy

FACTS:
Ang Giok Chip doing business under the name and style of
Hua Bee Kong Si was formerly the owner of a warehouse situated at No.
643 Calle Reina Regente, City of Manila. The contents of the warehouse
were insured with the three insurance companies for the total sum of
P60,000. One insurance policy, in the amount of P10,000, was taken out
with the Springfield Fire & Marine Insurance Company. The warehouse
was destroyed by fire on January 11, 1928, while the policy issued by the
latter company was in force.
The insurance company interposed its defense on a rider in
the policy in the form of Warranty F, fixing the amount of hazardous good
that can be stored in a building to be covered by the insurance. They
claimed that Ang violated the 3 percent limit by placing hazardous goods
to as high as 39 percent of all the goods stored in the building. His suit to
recover was granted by the trial court.

itself, because by the contract of insurance agreed to by the parties it


was made to be a part. It wasnt a separate instrument agreed to by the
parties.
The receipt of the policy by the insured without objection binds him. It
was his duty to read the policy and know its terms. He also never chose
to accept a different policy by considering the earlier one as a mistake.
PIONEER INSURANCE AND SURETY CORPORATION
vs.
OLIVA YAP

ISSUE:

FACTS: Respondent Oliva Yap was the owner of a store in a two-storey


building located at No. 856 Juan Luna Street, Manila, where in 1962 she
sold shopping bags and footwear, such as shoes, sandals and step-ins.
Chua Soon Poon Oliva Yap's son-in-law, was in charge of the store.
Yap took out a Fire Insurance Policy No. 4216 from Pioneer
Insurance with a value of P25,000.00 covering her stocks, office
furniture, fixtures and fittings.
Among the conditions in the policy executed by the parties are the
following:

Other insurance without the consent of Pioneer would avoid the contract.
It required no affirmative act of election on the part of the company to
make operative the clause avoiding the contract, wherever the specified
conditions should occur. Its obligations ceased, unless, being informed of
the fact, it consented to the additional insurance.
PIONEER

INSURANCE

&

SURETY

CORPORATION

vs.

unless such notice be given and the particulars of such insurance or


insurances be stated in, or endorsed on this Policy by or on behalf of the
Company before the occurrence of any loss or damage, all benefits
under this Policy shall be forfeited Any false declaration or breach or

THE HON. COURT OF APPEALS, BORDER MACHINERY & HEAVY


EQUIPMENT, INC., (BORMAHECO), CONSTANCIO M. MAGLANA and
JACOB S. LIM
FACTS:

this condition will render this policy null and void.


In 1965, Jacob S. Lim was engaged in the airline business as
owner-operator of Southern Air Lines (SAL) a single proprietorship.
Another insurance policy for P20,000.00 issued by Great
American covering the same properties. The endorsement recognized
co-insurance by Northwest for the same value.
Oliva Yap took out another fire insurance policy for P20,000.00 covering
the same properties from the Federal Insurance Company, Inc., which
was procured without notice to and the written consent of Pioneer.
A fire broke out in the building, and the store was burned. Yap filed an
insurance claim, but the same was denied for a breach.
Oliva Yap filed a case for payment of the face value of her fire insurance
policy. The insurance company refused to pay because she never
informed Pioneer of another insurer. The trial court decided in favor of
Yap. The CA affirmed.

On May 17, 1965, at Tokyo, Japan, Japan Domestic Airlines


(JDA) and Lim entered into and executed a sales contract (Exhibit A) for
the sale and purchase of two (2) DC-3A Type aircrafts and one (1) set of
necessary spare parts for the total agreed price of US $109,000.00 to be
paid in installments. One DC-3 Aircraft with Registry No. PIC-718,
arrived in Manila on June 7,1965 while the other aircraft, arrived in
Manila on July 18,1965.
On May 22, 1965, Pioneer Insurance and Surety Corporation
(Pioneer, petitioner in G.R. No. 84197) as surety executed and issued its
Surety Bond No. 6639 (Exhibit C) in favor of JDA, in behalf of its
principal, Lim, for the balance price of the aircrafts and spare parts.
On June 10, 1965, Lim doing business under the name and
style of SAL executed in favor of Pioneer as deed of chattel mortgage as
security for the latter's suretyship in favor of the former. It was stipulated
therein that Lim transfer and convey to the surety the two aircrafts. The
deed (Exhibit D) was duly registered with the Office of the Register of
Deeds of the City of Manila and with the Civil Aeronautics Administration
pursuant to the Chattel Mortgage Law and the Civil Aeronautics Law
(Republic Act No. 776), respectively.

ISSUE: Whether or not petitioner should be absolved from liability on


Fire Insurance Policy No. 4219 on account of any violation by
respondent Yap of the co-insurance clause therein.
RULING OF THE COURT:
There was a violation by respondent Oliva Yap of the coinsurance clause contained in Policy No. 4219 that resulted in the
avoidance of petitioner's liability.
The insurance policy for P20,000.00 issued by the Great
American, ceased to be recognized by them as a co-insurance policy.

The endorsement shows the clear intention of the parties to


recognize on the date the endorsement was made, the existence of only
one co-insurance, the Northwest one. The finding of the Court of Appeals
that the Great American Insurance policy was substituted by the Federal
Insurance policy is indeed contrary to said stipulation.

Lim defaulted on his subsequent installment payments


prompting JDA to request payments from the surety. Pioneer paid a total
sum of P298,626.12.

Pioneer then filed a petition for the extrajudicial foreclosure of


the said chattel mortgage before the Sheriff of Davao City. The
Cervanteses and Maglana, however, filed a third party claim alleging that
they are co-owners of the aircrafts,
On July 19, 1966, Pioneer filed an action for judicial
foreclosure with an application for a writ of preliminary attachment
against Lim and respondents, the Cervanteses, Bormaheco and
Maglana.
In their Answers, Maglana, Bormaheco and the Cervanteses
filed cross-claims against Lim alleging that they were not privies to the
contracts signed by Lim and, by way of counterclaim, sought for
damages for being exposed to litigation and for recovery of the sums of
money they advanced to Lim for the purchase of the aircrafts in question.
After trial on the merits, a decision was rendered holding Lim
liable to pay Pioneer but dismissed Pioneer's complaint against all other
defendants. The appellate court modified the trial court's decision in that
the plaintiffs complaint against all the defendants was dismissed

over "MV Asilda" together with her freight and appurtenances for the
purpose of limiting and extinguishing its liability under Art. 587 of the
Code of Commerce. 2
ISSUE:
Whether or not Pioneer is the real party in interest with regard
to the portion of the indemnity paid.

Issue:

RULING OF THE COURT:

Ruling of the Court: The court ruled in favor of PHILAMGEN.

The real party in interest with regard to the portion of the


indemnity paid is the insurer and not the insured. Petitioner was not the
real party in interest in the complaint therefore has no cause of action
against the respondents.

Interpreting the aforesaid provision, we ruled in the case of


Phil. Air Lines, Inc. v. Heald Lumber Co. (101 Phil. 1031 [1957]) which
we subsequently applied in Manila Mahogany Manufacturing
Corporation v. Court of Appeals(154 SCRA 650 [1987]):

Note that if a property is insured and the owner receives the


indemnity from the insurer, it is provided in said article that the insurer is
deemed subrogated to the rights of the insured against the wrongdoer
and if the amount paid by the insurer does not fully cover the loss, then
the aggrieved party is the one entitled to recover the deficiency.
Evidently, under this legal provision, the real party in interest with regard
to the portion of the indemnity paid is the insurer and not the insured.

It is clear from the records that Pioneer sued in its own name
and not as an attorney-in-fact of the reinsurer.

Case title: Philippine American General Insurance vs. Felman Shipping


Lines
Case number: GR 116940. June 11, 1997

WONPHILAMGEN was properly subrogated to the


rights and legal actions which the shipper had
against FELMAN, the ship-owner.

Discussion on how the SC ruled the case:

It is generally held that in every marine insurance policy the


assured impliedly warrants to the assurer that the vessel is seaworthy
and such warranty is as much a term of the contract as if expressly
written on the face of the policy. 12 Thus Sec. 113 of the Insurance Code
provides that "(i)n every marine insurance upon a ship or freight, or
freightage, or upon anything which is the subject of marine insurance, a
warranty is implied that the ship is seaworthy." Under Sec. 114, a ship is
"seaworthy when reasonably fit to perform the service, and to encounter
the ordinary perils of the voyage, contemplated by the parties to the
policy." Thus it becomes the obligation of the cargo owner to look for a
reliable common carrier which keeps its vessels in seaworthy condition.
He may have no control over the vessel but he has full control in the
selection of the common carrier that will transport his goods. He also has
full discretion in the choice of assurer that will underwrite a particular
venture.
The marine policy issued by PHILAMGEN to the Coca-Cola bottling firm
in at least two (2) instances has dispensed with the usual warranty of
worthiness. Paragraph 15 of the Marine Open Policy No. 100367-PAG
reads "(t)he liberties as per Contract of Affreightment the presence of the
Negligence Clause and/or Latent Defect Clause in the Bill of Lading
and/or Charter Party and/or Contract of Affreightment as between the
Assured and the Company shall not prejudice the insurance. The
seaworthiness of the vessel as between the Assured and the Assurers is
hereby admitted." 15

Facts:
Coca-Cola Bottlers Philippines, Inc., loaded on board "MV Asilda," a
vessel owned and operated by respondent Felman Shipping Lines
(FELMAN for brevity), 7,500 cases of 1-liter Coca-Cola soft drink bottles
to be transported from Zamboanga City to Cebu City for consignee
Coca-Cola Bottlers Philippines, Inc., Cebu. 1 The shipment was insured
with petitioner Philippine American General Insurance Co., Inc.
(PHILAMGEN for brevity), under Marine Open Policy No. 100367-PAG.
"MV Asilda" left the port of Zamboanga in fine weather at eight o'clock in
the evening of the same day. At around eight forty-five the following
morning, the vessel sank in the waters of Zamboanga del Norte bringing
down her entire cargo with her including the subject 7,500 cases of 1liter Coca-Cola soft drink bottles. The consignee Coca-Cola Bottlers
Philippines, Inc., Cebu plant, filed a claim with respondent FELMAN.
Respondent denied the claim thus prompting the consignee to file an
insurance claim with PHILAMGEN.Claiming its right of subrogation
PHILAMGEN sought recourse against respondent FELMAN which
disclaimed any liability for the loss. Consequently, PHILAMGEN sued the
ship owner.
Arguments:
In its complaint PHILAMGEN alleged that the sinking and total loss of
"MV Asilda" and its cargo were due to the vessel's unseaworthiness as
she was put to sea in an unstable condition. It further alleged that the
vessel was improperly manned and that its officers were grossly
negligent in failing to take appropriate measures to proceed to a nearby
port or beach after the vessel started to list.FELMAN filed a motion to
dismiss based on the affirmative defense that no right of subrogation in
favor of PHILAMGEN was transmitted by the shipper, and that, in any
event, FELMAN had abandoned all its rights, interests and ownership

Having disposed of this matter, we move on to the legal basis for


subrogation. PHILAMGEN's action against FELMAN is squarely
sanctioned by Art. 2207 of the Civil Code which provides:
Art. 2207. If the plaintiff's property has been insured,
and he has received indemnity from the insurance
company for the injury or loss arising out of the
wrong or breach of contract complained of, the
insurance company shall be subrogated to the rights
of the insured against the wrongdoer or the person
who has violated the contract. If the amount paid by
the insurance company does not fully cover the
injury or loss, the aggrieved party shall be entitled to
recover the deficiency from the person causing the
loss or injury.
The doctrine of subrogation has its roots in equity. It is designed to
promote and to accomplish justice and is the mode which equity adopts
to compel the ultimate payment of a debt by one who in justice, equity
and good conscience ought to pay. 19 Therefore, the payment made by
PHILAMGEN to Coca-Cola Bottlers Philippines, Inc., gave the former the
right to bring an action as subrogee against FELMAN. Having failed to
rebut the presumption of fault, the liability of FELMAN for the loss of the
7,500 cases of 1-liter Coca-Cola soft drink bottles is inevitable.
Case Title: Commissioner vs. Lincoln
Case Number: G.R. No. 119176

March 19, 2002

Facts: Private respondent Lincoln Philippine Life Insurance Co., Inc.,


(now Jardine-CMA Life Insurance Company, Inc.) is a domestic
corporation registered with the Securities and Exchange Commission
and engaged in life insurance business. In the years prior to 1984,
private respondent issued a special kind of life insurance policy known
as the "Junior Estate Builder Policy," the distinguishing feature of which
is a clause providing for an automatic increase in the amount of life
insurance coverage upon attainment of a certain age by the insured
without the need of issuing a new policy. The clause was to take effect in
the year 1984. Documentary stamp taxes due on the policy were paid by
petitioner only on the initial sum assured.

obligation,9 but still a part of the insurance sold to which private


respondent was liable for the payment of the documentary stamp tax.

Documentary stamp taxes were paid based only on the par value
of P5,000,000.00 and not on the book value.

Case Number: [GR 119655, 24 Ma y 1996]

Subsequently, petitioner issued deficiency documentary


stamps tax assessment for the year 1984 corresponding to the
amount of automatic increase of the sum assured on the policy
issued by respondent, and (corresponding to the book value in
excess of the par value of the stock dividends. Private
respondent questioned the deficiency assessments and
sought their cancellation in a petition filed in the Court of Tax
Argument: Petitioner claims that the "automatic increase clause" in the
subject insurance policy is separate and distinct from the main
agreement and involves another transaction; and that, while no new
policy was issued, the original policy was essentially re-issued when the
additional obligation was assumed upon the effectivity of this "automatic
increase clause" in 1984; hence, a deficiency assessment based on the
additional insurance not covered in the main policy is in order.

Issue: THE HONORABLE COURT OF APPEALS ERRED WHEN IT


RULED THAT THERE IS A SINGLE AGREEMENT EMBODIED IN THE
POLICY AND THAT THE AUTOMATIC INCREASE CLAUSE IS NOT A
SEPARATE AGREEMENT, CONTRARY TO SECTION 49 OF THE
INSURANCE CODE AND SECTION 183 OF THE REVENUE CODE
THAT A RIDER, A CLAUSE IS PART OF THE POLICY.
Ruling: The petition is impressed with merit.
Discussion on how the SC ruled the case:
The subject insurance policy at the time it was issued contained an
"automatic increase clause." Although the clause was to take effect only
in 1984, it was written into the policy at the time of its issuance. The
distinctive feature of the "junior estate builder policy" called the
"automatic increase clause" already formed part and parcel of the
insurance contract, hence, there was no need for an execution of a
separate agreement for the increase in the coverage that took effect in
1984 when the assured reached a certain age.
Here, although the automatic increase in the amount of life insurance
coverage was to take effect later on, the date of its effectivity, as well as
the amount of the increase, was already definite at the time of the
issuance of the policy. Thus, the amount insured by the policy at the time
of its issuance necessarily included the additional sum covered by the
automatic increase clause because it was already determinable at the
time the transaction was entered into and formed part of the policy.
The "automatic increase clause" in the policy is in the nature of a
conditional obligation under Article 1181,8 by which the increase of the
insurance coverage shall depend upon the happening of the event which
constitutes the obligation. In the instant case, the additional insurance
that took effect in 1984 was an obligation subject to a suspensive

The deficiency of documentary stamp tax imposed on private respondent


is definitely not on the amount of the original insurance coverage, but on
the increase of the amount insured upon the effectivity of the "Junior
Estate Builder Policy."
Case Title: Tib a y vs. Cou rt of Ap pe als

Facts:
On 22 Januar y 1987, For tune Life and Genera l
In surance Co., In c. (For tune) issued Fire Insuran ce Policy
136171 in favor of Violeta R. Tibay and/or Nicolas Roraldo on their twostorey residential building located at 5855 Zobel Street, Makati City,
together with all their personal effects therein. The insurance was for
P600, 000.00 covering the period from 23 January 1987 to 23 January
1988. On 23 January 1987, of the total premium of P2,983.50, petitioner
Violeta Tibay only paid P600.00 thus leaving a considerable balance
unpaid. On 8 March 1987 the insured build ing wa s
comple tel y destro ye d by fire . Two days late r Violeta Tibay paid
the balance of the premium. On the same day, she filed with Fortune a
claim on the fire insurance policy. Her claim was accordingly referred
to its adjuster, Goodwill Adjustment Services,Inc. (GASI), which
immediately wrote Violeta requesting her to furnish it with the necessary
documents for the investigation and processing of her claim. Petitioner
forthwith complied. On 28 March 1987 she signed a non-waiver
agreement with GASI to the effect that any action taken by the
companies or their representatives in investigating the claim made by the
claimant for his loss which occurred at 5855 Zobel Roxas, Makati on
8March 1987, or in the investigating or ascertainment of the amount of
actual cash value and loss, shall not waive or invalidate any condition of
the policies of such companies held by said claimant, nor the rights of
either or any of the parties to this agreement, and such action shall not
be, or be claimed to be, an admission of liability on the part of said
companies or any of them. In a letter Fortune denied the claim of Violeta
for violation of Policy Condition 2 and of Section 77 of the Insurance
Code. Efforts to settle the case before the Insurance Commission proved
futile. Violeta and the other petitioners( Antonio Tibay, Ofel ia M .
Roraldo, Victo rina M. Roraldo , Virgil io M. Roraldo, M yr na M .
Roraldo, and Rosabella M . Rora ldo) sued For tune for
damages
in
the
amoun t
of
the
total
coverage of the fire insurance policy plus 12% inter
est per annum, P100,000.00 moral damages, and
attorney's fees equivalent to 20% of the total claim

Argument:
Petitioners maintain otherwise. Insisting that
FORTUNE is liable on the policy despite partial payment of the premium
due and the express stipulation thereof to the contrary, petitioners rely
heavily on the 1967 case of Philippine Phoenix and Insurance Co.,
Inc. v. Woodworks, Inc. 8 where the Court through Mr. Justice Arsenio P.
Dizon sustained the ruling of the trial court that partial payment of the
premium made the policy effective during the whole period of the policy.
In that case, the insurance company commenced action against the
insured for the unpaid balance on a fire insurance policy. In its defense
the insured claimed that nonpayment of premium produced the
cancellation of the insurance contract.

Issue: W hether a fire insurance poli cy be valid, bindin g and


enforceable upon mere partia l paymen t of premium..

i.

Facts:

Canilang consulted Dr. Claudio and was diagnosed as suffering from


"sinus tachycardia." Mr. Canilang consulted the same doctor again on 3
Held:

The SC answered in the negative

August 1982 and this time was found to have "acute bronchitis."

Discussion on how the SC ruled the case:


On the next day, 4 August 1982, Canilang applied for a "non-medical"
insurance policy with Grepalife naming his wife, as his beneficiary.
Insurance is a contract whereby one undertakes for a
consideration to indemnify another against loss, damage or liab ili ty
ari sing from an unkno wn or contingent event. The
con sidera tion i s the prem ium, wh ich must be paid at the
time and in the wa y and manner specified in the poli cy, and
if not so paid , the policy wil l lapse and be forfeited b y its
o wn term s. T he Po licy provides for pa ym ent of premium in
full . Accordingly, where the premium has only been partially paid and
the balance paid only after the peril insured a gain st has occur red,
the insuran ce contract did no t ta ke effect and the insu red
canno t co llect a t all on the policy. This is fully supported by Section
77 of the Insurance Code which provides that "An insurer is entitled
to payment of the premium as soon as the thi
ng insured is exposed to the peril insured ag
a i n s t . Notwi th standing an y a greemen t to the con trar y, no
poli cy or con tra ct of in surance issued by an insurance
company is valid and binding unless and until the premium thereof has
been
paid,
except
in
the
case
of
a
life
or an industrial life policy whenever the grace perio
d provision applies." Apparently the crux of the
con troversy lie s in the phrase "unle ss and un til the premium
thereof has been paid ." This lead s us to the manner of
payment envisioned by the law to make the insurance policy operative
and binding. For whatever judicial construction may be accorded the
disputed phrase must ultimately yield to the clear mandate of the law.
The principle that where the law does not distinguish the court should
neither distinguish assumes that the legislature made no qualification on
the use of a general word or expression and it cannot be disputed that
premium is the elixir vitae of the insurance business because by law the
insurer must maintain a legal reserve fund to meet its contingent
obligations to the public, hence, the imperative need for its prompt
payment
and
full satisfaction. It must be emphasized here that al
l actuarial calculations and various tabulations of
probabilities of losses under the risks insured against are based on
the sound hypothesis of prompt payment of premiums. Upon this
bedrock insurance firms are enabled to offer the assurance of security to
the public at favorable ra tes. But once pa ymen t of premium is
lef t to the whi m and caprice of the insu red, as wh en the
courts tolerate the payment of a mere P600.00 as partial undertaking out
of the stipulated total premium ofP2,983.50 and the balance to be paid
even after the risk insured against has occurred, as Tibay et al. have
done in this case, on the principle that the strength of the vinculum juris
is not measured by any specific amount of premium payment, we will
surely wreak havoc on the business and set to naught what has taken
actuarians centuries to devise to arrive at a fair and equitable distribution
of risks and benefits between the insurer and the insured.

Canilang was issued ordinary life insurance with the face value of
P19,700.
On 5 August 1983, Canilang died of "congestive heart failure,"
"anemia," and "chronic anemia." The wife as beneficiary, filed a claim
with Grepalife which the insurer denied on the ground that the insured
had concealed material information from it.
Argument:

Vda Canilang filed a complaint with the Insurance

Commissioner against Grepalife contending that as far as she knows her


husband was not suffering from any disorder and that he died of kidney
disorder.
Grepalife was ordered to pay the widow by the Insurance
Commissioner holding that there was no intentional concealment on the
Part of Canilang and that Grepalife had waived its right to inquire into the
health condition of the applicant by the issuance of the policy despite the
lack of answers to "some of the pertinent questions" in the insurance
application. CA reversed.

ii.

Issue:

Whether or not Grepalife is liable.


Held:

Petition lack of merit

Discussion on how the SC ruled the case:


iii.
SC took note of the fact that Canilang failed to disclose that hat he had
twice consulted Dr. Wilfredo B. Claudio who had found him to be
suffering from "sinus tachycardia" and "acute bronchitis. Under the
relevant provisions of the Insurance Code, the information concealed
must be information which the concealing party knew and "ought to
[have] communicate[d]," that is to say, information which was "material to

Case Title: Canilang vs. Great Pacific

the contract.

Case Number: GR No. 92492


The information which Canilang failed to disclose was material to the
ability of Grepalife to estimate the probable risk he presented as a
subject of life insurance. Had Canilang disclosed his visits to his doctor,

the diagnosis made and the medicines prescribed by such doctor, in the
insurance application, it may be reasonably assumed that Grepalife
would have made further inquiries and would have probably refused to
issue a non-medical insurance policy or, at the very least, required a
higher premium for the same coverage.

The materiality of the information withheld by Canilang from Grepalife did

RESPONDENT:
The defendant denied the claim, on the ground they
defendant's investigation revealed that the entire shipment of logs
covered by the two marines policies No. 53 110 1032 and 713 HO 1033
were received in good order at their point of destination. It was further
stated that the said loss may be considered as covered under Cover
Note No. 1010 because the said Note had become 'null and void by
virtue of the issuance of Marine Policy Nos. 53 HO 1032 and 1033.
PETITIONER:
Petitioner contendd that the Cover Note was issued
with a consideration when, by express stipulation, the cover note is
made subject to the terms and conditions of the marine policies, and the
payment of premiums is one of the terms of the policies.

not depend upon the state of mind of Jaime Canilang. A man's state of
mind or subjective belief is not capable of proof in our judicial process,

ISSUE:

Whether the Note is to be treated as a separate policy?

except through proof of external acts or failure to act from which


inferences as to his subjective belief may be reasonably drawn. Neither
does materiality depend upon the actual or physical events which ensue.
Materiality relates rather to the "probable and reasonable influence of the
facts" upon the party to whom the communication should have been
made, in assessing the risk involved in making or omitting to make
further inquiries and in accepting the application for insurance; that

RULING: It is not disputed that petitioner paid in full all the premiums as
called for by the statement issued by private respondent after the
issuance of the two regular marine insurance policies, thereby leaving no
account unpaid by petitioner due on the insurance coverage, which must
be deemed to include the Cover Note. If the Note is to be treated as a
separate policy instead of integrating it to the regular policies
subsequently issued, the purpose and function of the Cover Note would
be set at naught or rendered meaningless, for it is in a real sense a
contract, not a mere application for insurance which is a mere offer.

"probable and reasonable influence of the facts" concealed must, of


course, be determined objectively, by the judge ultimately.

SC found it difficult to take seriously the argument that Grepalife had


waived inquiry into the concealment by issuing the insurance policy
notwithstanding Canilang's failure to set out answers to some of the
questions in the insurance application. Such failure precisely constituted

DISCUSSION:
It may be true that the marine insurance policies
issued were for logs no longer including those which had been lost
during loading operations. This had to be so because the risk insured
against is not for loss during operations anymore, but for loss during
transit, the logs having already been safely placed aboard. This would
make no difference, however, insofar as the liability on the cover note is
concerned, for the number or volume of logs lost can be determined
independently as in fact it had been so ascertained at the instance of
private respondent itself when it sent its own adjuster to investigate and
assess the loss, after the issuance of the marine insurance policies.

concealment on the part of Canilang. Petitioner's argument, if accepted,


would obviously erase Section 27 from the Insurance Code of 1978.
PACIFIC BANKING CORPORATION vs. COURT OF APPEALS and
ORIENTAL ASSURANCE CORPORATION
PACIFIC TIMBER EXPORT CORPORATION vs. CA and WORKMEN'S
INSURANCE COMPANY, INC.
G.R. No. L-38613

February 25, 1982 DE CASTRO, J.:

Case # 57
FACTS: The plaintiff secured temporary insurance from the defendant
for its exportation of 1,250,000 board feet logs.The defendant issued
Cover Note No. 1010, insuring the said cargo of the plaintiff. The regular
marine cargo policies were issued by the defendant in favor of the
plaintiff. The two marine policies bore the numbers 53 HO 1032 and 53
HO 1033. After the issuance of Cover Note No. 1010, but before the
issuance of the two marine policies Nos. 53 HO 1032 and 53 HO 1033,
some of the logs intended to be exported were lost during loading
operations.
The plaintiff subsequently submitted a 'Claim Statement
demanding payment of the loss under Policies Nos. 53 HO 1032 and 53
HO 1033.

ARGUMENTS:

G.R. No. L-41014

November 28, 1988 PARAS, J.:

Case # 58
FACTS: Fire Policy, an open policy, was issued to the Paramount Shirt
Manufacturing Co. by which private respondent Oriental Assurance
Corporation bound itself to indemnify the insured for any loss or damage
caused by fire to its property.
Said policy was duly endorsed to petitioner as mortgagee/ trustor of the
properties insured, with the knowledge and consent of private
respondent to the effect that "loss if any under the policy is payable to
the Pacific Banking Corporation".
While the aforesaid policy was in full force and effect, a fire broke out on
the subject premises destroying the goods . Petitioner sent a letter of
demand to private respondent for indemnity .
At the trial, petitioner presented evidence, a communication of the
insurance adjuster to Asian Surety Insurance Co., Inc., revealing
undeclared co-insurances with the following: with Wellington Insurance;
with Empire Surety and with Asian Surety; undertaken by insured
Paramount on the same property covered by its policy with private

respondent whereas the only co-insurances declared in the subject


policy are those of with Malayan, with South Sea and with Victory.
ARGUMENTS:
RESPONDENT:
Private respondent raised the defense of fraud
and/or violation of Condition No. 3 in the Policy, in the form of nondeclaration of co-insurances.
PETITIONER:
suspicion.

The allegation of fraud is but a mere inference or

ISSUE:

Whether the plaintiff if guilty of fraud?

RULING: It is not disputed that the insured failed to reveal before the
loss three other insurances. By reason of said unrevealed insurances,
the insured had been guilty of a false declaration; a clear
misrepresentation and a vital one because where the insured had been
asked to reveal but did not, that was deception. Otherwise stated, had
the insurer known that there were many co-insurances, it could have
hesitated or plainly desisted from entering into such contract. Hence, the
insured was guilty of clear fraud
DISCUSSION:
Concrete evidence of fraud or false declaration by
the insured was furnished by the petitioner itself when the facts alleged
in the policy under clauses "Co-Insurances Declared" and "Other
Insurance Clause" are materially different from the actual number of coinsurances taken over the subject property. Consequently, "the whole
foundation of the contract fails, the risk does not attach and the policy
never becomes a contract between the parties. Representations of facts
are the foundation of the contract and if the foundation does not exist,
the superstructure does not arise. Falsehood in such representations is
not shown to vary or add to the contract, or to terminate a contract which
has once been made, but to show that no contract has ever existed.
TAN vs. CA and THE PHILIPPINE AMERICAN LIFE INSURANCE
COMPANY
G.R. No. 48049

June 29, 1989

GUTIERREZ, JR., J.:

The so-called "incontestability clause" precludes the insurer


from raising the defenses of false representations or concealment of
material facts insofar as health and previous diseases are concerned if
the insurance has been in force for at least two years during the
insured's lifetime. The phrase "during the lifetime" found in Section 48
simply means that the policy is no longer considered in force after the
insured has died. The key phrase in the second paragraph of Section 48
is "for a period of two years."
DISCUSSION:
The insurer has two years from the date of issuance
of the insurance contract or of its last reinstatement within which to
contest the policy, whether or not, the insured still lives within such
period. After two years, the defenses of concealment or
misrepresentation, no matter how patent or well founded, no longer lie.
Congress felt this was a sufficient answer to the various tactics employed
by insurance companies to avoid liability. The petitioners' interpretation
would give rise to the incongruous situation where the beneficiaries of an
insured who dies right after taking out and paying for a life insurance
policy, would be allowed to collect on the policy even if the insured
fraudulently concealed material facts.
QUA CHEE GAN vs. LAW UNION AND ROCK INSURANCE CO., LTD.,
represented by its agent, WARNER, BARNES AND CO., LTD.,
G.R. No. L-4611

December 17, 1955

REYES, J. B. L., J.:

Case no. 60
FACTS: Qua Chee Gan owned four warehouses or bodegas used for
the storage of stocks of copra and of hemp, baled and loose, in which
the appellee dealth extensively. They had been, with their contents,
insured with the defendant Company.
Fire of undetermined origin that broke out and lasted almost
one week, gutted and completely destroyed Bodegas Nos. 1, 2 and 4,
with the merchandise stored theren. Plaintiff-appellee informed the
insurer by telegram, the fire adjusters engaged by appellant insurance
company arrived and proceeded to examine and photograph the
premises, pored over the books of the insured and conducted an
extensive investigation. The plaintiff having submitted the corresponding
fire claims, the Insurance Company resisted payment.

Case # 59
FACTS: Tan Lee Siong, applied for life insurance with respondent
company, with petitioners the beneficiaries thereof. Tan Lee Siong died
of hepatoma. Petitioners then filed with respondent company their claim
for the proceeds of the life insurance policy. However, respondent
company denied petitioners' claim and rescinded the policy .
ARGUMENTS:
RESPONDENT:
Alleged misrepresentation and concealment of
material facts made by the deceased Tan Lee Siong in his application for
insurance.
PETITIONERS:
Contend that the respondent company no longer
had the right to rescind the contract of insurance as rescission must
allegedly be done during the lifetime of the insured within two years and
prior to the commencement of action.

ISSUE:

Whether the contention of the petitioners is tenable?

RULING: Sec. 48. xxx After a policy of life insurance made payable on
the death of the insured shall have been in force during the lifetime of the
insured for a period of two years from the date of its issue or of its last
reinstatement, the insurer cannot prove that the policy is void ab initio or
is rescindable by reason of the fraudulent concealment or
misrepresentation of the insured or his agent.

ARGUMENTS:
RESPONDENT:
warranty.

Claims violation of the so-called fire hydrants

PETITIONER: Claims that the respondent is estopped for failure to reject


the application due to defect of warranty which was existing at the
inception of the insurance contract.
ISSUE:

Whether the insurer is liable?

RULIING: The appellant is barred by waiver (or rather estoppel) to claim


violation of the so-called fire hydrants warranty, for the reason that
knowing fully all that the number of hydrants demanded therein never
existed from the very beginning, the appellant neverthless issued the
policies in question subject to such warranty, and received the
corresponding premiums. It would be perilously close to conniving at
fraud upon the insured to allow appellant to claims now as void ab initio
the policies that it had issued to the plaintiff without warning of their fatal
defect, of which it was informed, and after it had misled the defendant
into believing that the policies were effective.

DISCUSSION:
It is usually held that where the insurer, at the time
of the issuance of a policy of insurance, has knowledge of existing facts
which, if insisted on, would invalidate the contract from its very inception,
such knowledge constitutes a waiver of conditions in the contract
inconsistent with the facts, and the insurer is estopped thereafter from
asserting the breach of such conditions. The law is charitable enough to
assume, in the absence of any showing to the contrary, that an insurance

company intends to executed a valid contract in return for the premium


received; and when the policy contains a condition which renders it
voidable at its inception, and this result is known to the insurer, it will be
presumed to have intended to waive the conditions and to execute a
binding contract, rather than to have deceived the insured into thinking
he is insured when in fact he is not, and to have taken his money without
consideration
Case no. 64
G.R. No. L-50997 June 30, 1987
SUMMIT GUARANTY AND INSURANCE COMPANY, INC., petitioner,
vs.
HON. JOSE C. DE GUZMAN, in his capacity as Presiding Judge of
Branch III, CFI of Tarlac, GERONIMA PULMANO and ARIEL
PULMANO, respondents.
No. L-48679 June 30, 1987
SUMMIT GUARANTY AND INSURANCE COMPANY, INC., petitioner,
vs.
THE HONORABLE GREGORIA C. ARNALDO, in her capacity as
Insurance Commissioner, and JOSE G. LEDESMA, JR.,
respondents.
No. L-48758 June 30, 1987
SUMMIT GUARANTY AND INSURANCE COMPANY, INC., petitioner,
vs.
HONORABLE RAMON V. JABSON, in his capacity as Presiding
Judge of Branch XXVI, Court of First Instance of Rizal, Pasig, Metro
Manila and AMELIA GENERAO, respondents.
Facts: Jose Ledesma was the owner of a tractor which was bumped by
a minibus insured with petitioner for Third Party Liability. Ledesma
immediately made a notice of claim. Petitioner company advised private
respondent to have car repaired by G.A. Machineries, which was later
estimated at an amount of Php21,000 and made assurance of payment.
Upon repair, respondent made several demands on insurance company
because of repair shops warning that failure to pay would result in the
auctioning of the tractor to pay expenses. Petitioner Company continued
giving assurance and promises to pay. Eventually, private respondent
filed a formal complaint with the Insurance Commission, which petitioner
company moved to dismiss on ground of prescription.
Geronima Pulmano was the owner of a jeep insured with petitioner
company in the amount of Php20,000. The jeep got into a vehicular
accident which resulted in the death of one of the victims and private
respondent immediately filed a notice of accident and claim. Petitioner
company took no steps to process the claim so private respondents
brought their claim to the Insurance Commission, but petitioner company
still failed to settle. A complaint was eventually filed with the Court of First
Instance of Tarlac which petitioner company moved to dismiss on the
ground of prescription.
Amelia Generao owned a passenger jeepney insured with petitioner
under a Vehicle Comprehensive Policy. The jeepney struck the van of a
certain Mr. Hahn and two days later Generao notified insurance
company and demanded payment on both vehicles. Generao and
petitioner insurance company even had a dialogue at the office of
insurance company to settle the claim. Nonetheless, time passed without
petitioner insurance company taking any final action. Mr. Hahn filed a
complaint for damages against Generao who, in response, filed a third
party complaint against petitioner insurance company which in turn filed
a motion to dismiss on the ground of prescription.
Petitioner insurance Company argues that under Section 384 of the
Insurance Code, even if the notice of claim was timely filed with the
insurance company within the six month period, if the action or suit that
follows is filed beyond the one year period it should necessarily be
dismissed on the ground of prescription.

settlement of claims, and with manifest bad faith, petitioner Company


devised means and ways of stalling the settlement proceedings.
The one year period should be counted from the date of rejection by the
insurer as this is the time the cause of action accrues. Since in these
cases there has yet been no accrual of cause of action, prescription has
not yet set in.
Section 384 has been amended as follows, Action or suit for recovery
of damage due to loss or injury must be brought in proper cases, with the
Commissioner or the Courts within one year from denial of the claim,
otherwise the claimant right of action shall prescribe.

Bonifacio Bros., Inc., et al. vs Enrique Mora, et al.


GR No. L-20853, May 29, 1967

Facts:
Enrique Mora, owner of an Oldsman sedan mortgaged the same to the
HS Reyes Inc. Thereafter, the automobile was insured with the State
Bonding & Insurance Co., Inc with the provision that Loss, if any, is
payable to HS Reyes Inc. by virtue of the fact that said Oldsman sedan
was mortgaged in favor of the latter.

During the effectivity of the insurance contract, the car met with an
accident. Enrique Mora, without the knowledge and consent of HS
Reyes Inc. authorized Bonifacio Bros. Inc. to furnish the labor and
materials and some of which were supplied by the Ayala Auto Parts Co.
Proceeds of the insurance policy was not given to Bonifacio Bros. Inc.
and Ayala Auto Parts Co., hence, complaint was filed before Municipal
Court of Manila against Enrique Mora and the insurance company for the
labor and materials supplied.

The appellants argued that they are privy to the contract. On the other
hand, the insurance company maintains that appellants are not
mentioned in the contract as parties thereto nor is there any clause or
provision from which it can be inferred that there is an obligation on the
part of the insurance company to pay the cost of repairs directly to them.
Issue: Whether there is privity of contract between the Bonifacio Bos.
Inc and the Ayala Auto Parts Co. on the one hand and the insurance
company on the other.
Ruling:
The appellants are not privy to the contract, hence, they have no right of
action against the insurance company.

A policy of insurance is a distinct and independent contract between the


insured and insurer, and third persons have no right either in a court of
equity, or in a court of law, to the proceeds of it, unless there be some
contract of trust, expressed or implied, by the insured and the third
person.

Issue: Whether or not the causes of action of private respondents have


already prescribed.
Ruling: NO. The Supreme Court finds absolutely nothing in the law
which mandates that the two periods must always concur. On the
contrary, it is very clear that the one year period is only required in
proper cases. It is very obvious that petitioner Company is trying to use
Section 384 of as a cloak to hide itself from its liabilities. In violation of its
duties to adopt and implement reasonable standards for the prompt
investigation of claims and to effectuate prompt, fair and equitable

It is fundamental that contracts take effect only between the parties


thereto, except on some specific cases provided by law where the
contract contains some stipulation in favor of a third person (Art. 1311,
Civil Code) Such stipulation is known as stipulation pour autrui or a
provision in favor of a third person not party to the contract. In the instant
case, the insurance contract does not contain any words or clauses to
disclose an intent to give any benefit to any repairmen or material men in

case of repair of the car in question. The loss payable clause of the
insurance policy stipulates that Loss, if any, is payable to HS Reyes,
Inc indicating that it was only the HS Reyes Inc. which they intended to
benefit.
Malayan Insurance Co., Inc (MICO) vs Gregoria Cruz Arnaldo and
Coronacion Pinca

that the cancellation was actually sent to and received by the insured.
Adora, incidentally, had not been informed of the cancellation either and
saw no reason not to accept the said payment. As to the authority of
Adora to receive payment, Payment to an agent having the authority to
receive or collect payment is equivalent to payment to the principal
himself; such payment is complete when the money delivered is into the
agents hand and is a discharge of the indebtedness owing to the
principal

G.R. No. 67835. October 12, 1987


Facts:
Petition denied. The decision of the Insurance Commission affirmed.
On June 7, 1981, petitioner MICO issued to the private respondent,
Coronacion Pinca, Fire Insurance Policy on her property effective July
22, 1981 until July 22, 1982.
Ng Gan Zee vs Asian Crusader Life Insurance
On October 15, 1981, MICO allegedly cancelled the policy for nonpayment of the premium and sent the corresponding notice to Pinca.

GR No. L-30685, may 30, 1983

On December 24, 1981, payment of the premium for Pinca was received
by Domingo Adora, agent of MICO.
Facts:
On January 15, 1982, Adora remitted this payment to MICO, together
with other payments.
On January 18, 1982, Pincas property was completely burned.
On February 5, 1982, Pincas payment was returned by MICO to Adora
on the ground that her policy had been cancelled earlier but Adora
refused to accept.

Pinca made demands for payment but MICO rejected. Such demand
was sustained by the respondent Insurance Commission, hence this
petition. The petitioner argues that there was no payment of premium
and that the policy had been cancelled before the occurrence of the loss.
Also, Adora was not authorized to accept the premium payment because
six months had elapsed since the issuance of the insurance policy and
such acceptance was prohibited by the policy itself. It is also argued that
this prohibition was binding upon Pinca, who made the payment to Adora
at her own risk as she was bound to first check his authority to receive it.

On May 12, 1962, Kwong Nam applied for a 20-year endowment insurance on his life
for the sum of P20,000.00, with his wife, appellee Ng Gan Zee, as beneficiary. On the
same date, appellant, upon receipt of the required premium from the insured, approved
the application and issued the corresponding policy.

Upon Kwong Nams death due to cancer of the liver with metastasis, appellant denied
the claim on the ground that the answers given by the insured to the questions
appearing in his application for life insurance were untrue. Appellant further maintain
that when the insured was examined in connection with his application for life insurance
he gave the appellants medical examiner false and misleading information as to his
aliment and previous operation. Appellant argues that the insureds statement in hi
application that a tumor hard and of a hens egg size was removed during said
operation, constituted material concealment.

Issue:
Issue: Whether there exists an insurance contract at the time of the loss
sustained by Pinca.

Whether the insurance company, because of the insureds representation, was mislead
or deceived into entering the contract.
Ruling:
Yes. There exists a valid contract of insurance.

Ruling:

Payment of premium was in fact made, rendering the policy effective as


of June 22, 1981, and removing it from the provisions of Sec 77.- An
insurer is entitled to payment of the premium as soon as the thing is
exposed to the peril insured against. Notwithstanding any agreement to
the contrary, no policy or contract of insurance issued by an
insurance company is valid and binding unless and until the
premium thereof has been paid, except in the case of a life or an
industrial life policy whenever the grace period provision applies.

No. It bears emphasis that Kwong Nam had informed the appellants medical examine
that the tumor for which he was operated on was associated with ulcer of the stomach
and in the absence of evidence that the insured had sufficient medical knowledge as to
enable him to distinguish between peptic ulcer and a tumor, his statement should be
construed as an expression made in good faith of his belief as to the nature of hi
ailment and operation. Thus, concealment exists where the assured had knowledge o
a fact material to the risk, and honesty, good faith and fair dealing requires that he
should communicate it to the assurer, but he designedly and intentionally withholds the
same. Indeed kwong Nams statement must be presumed to have been made by him
without knowledge of its incorrectness and without any deliberate intent on his part to
mislead the appellant.

As to the claim of MICO that it cancelled the policy in question on


October 15, 1981, for non-payment of premium, there is a flat denial of
Pinca that she never received the claimed cancellation and who, of
course, did not have to prove such denial considering the strict language
of Section 64 of the Insurance Law that no insurance policy shall be
cancelled except upon prior notice. MICO has the burden in making sure

Also, it has been held that where, upon the face of the application, a question appear
to be not answered at all or to be imperfectly answered, and the insurer issued a polic

without any further inquiry, they waive the imperfection of the answer and render the
omission to answer more fully immaterial.
Ruling:
The Philippine American Life Insurance Co. vs Hon. Gregorio
Pineda and Dimayuga
G.R. No. 54216. July 19, 1989

Facts:

In 1968, private respondent procured an ordinary life insurance policy


from the petitioner company and designated his wife and children as
irrevocable beneficiaries of said policy. In 1980, the petitioner
subsequently filed a petition before the CFI of Rizal to amend the
designation of the beneficiaries in his life policy from irrevocable to
revocable. Respondent Judge Pineda allowed the private respondent to
adduce evidence. The argument of the petitioner is the designation of
the beneficiaries in the policy has been made without reserving the right
to change said beneficiary/beneficiaries, such designation may not be
surrendered to the Company, released or assigned; and no right or
privilege under the Policy may be exercised, or agreement made with the
Company to any change in or amendment to the Policy, without the
consent of the said beneficiary/beneficiaries. On the other hand, the
private respondent contends that said designation can be amended if the
Court finds a just, reasonable ground to do so.

Issue: Whether the designation of the irrevocable beneficiaries could be


changed or amended without the consent of all the irrevocable
beneficiaries.

No. The applicable law in the instant case is the Insurance Act,
otherwise known as Act No. 2427 as amended, the policy having been
procured in 1968. Under the said law, the beneficiary designated in a life
insurance contract cannot be changed without the consent of the
beneficiary because he has a vested interest in the policy (Gercio v. Sun
Life Ins. Co. of Canada, 48 Phil. 53; Go v. Redfern and the International
Assurance Co., Ltd., 72 Phil. 71).

The Beneficiary Designation Indorsement in the policy of the respondent


states that the designation of the beneficiaries is irrevocable, hence,both
the law and the policy do not provide for any other exception. Such fact
was not disproved by the respondent.

The contract between the parties is the law binding on both of them and
for so many times, this court has consistently issued pronouncements
upholding the validity and effectivity of contracts. Where there is nothing
in the contract which is contrary to law, good morals, good customs,
public policy or public order the validity of the contract must be
sustained. Likewise, contracts which are the private laws of the
contracting parties should be fulfilled according to the literal sense of
their stipulations, if their terms are clear and leave no room for doubt as
to the intention of the contracting parties, for contracts are obligatory, no
matter in what form they may be, whenever the essential requisites for
their validity are present (Phoenix Assurance Co., Ltd. vs. United States
Lines, 22 SCRA 675, Phil. American General Insurance Co., Inc. vs.
Mutuc, 61 SCRA 22.)

Você também pode gostar